Tải bản đầy đủ (.doc) (83 trang)

Bất đẳng thức (Chuyên đề Bồi dưỡng HSG) - Tài liệu Toán 9

Bạn đang xem bản rút gọn của tài liệu. Xem và tải ngay bản đầy đủ của tài liệu tại đây (1.27 MB, 83 trang )

<span class='text_page_counter'>(1)</span><div class='page_container' data-page=1>

<b>Phần I - kiến thức cơ bản</b>
<b>I . Một số bất đẳng thức cần nhớ:</b>


2 <sub>0;</sub> <sub>0;</sub>


<i>a</i>  <i>a</i>   <i>b</i>  <i>b b</i>


o Bất đẳng thức Cơ sy:


<i>n</i>
<i>n</i>


<i>n</i> <i><sub>a</sub><sub>a</sub></i> <i><sub>a</sub></i> <i><sub>a</sub></i>
<i>n</i>


<i>a</i>
<i>a</i>


<i>a</i>
<i>a</i>


....
....


3
2
1
3


2
1









Víi <i>a<sub>i</sub></i> 0


dấu bằng xảy ra khi

<i>a</i>

<sub>1</sub>

<i>a</i>

<sub>2</sub>

 

...

<i>a</i>

<i><sub>n</sub></i>
o Bất đẳng thức Bunhiacopski:


 

<sub>2</sub>

2

<i>aa</i>

<sub>2</sub>

2



<i><sub>n</sub></i>

2

...

 

<sub>1</sub>

2



<i>xxa</i>

<sub>2</sub>

2

....



2

<i><sub>n</sub></i>

 

<sub>1</sub>



<i>axa</i>

<sub>221</sub>

....



<i>xax</i>

<i><sub>nn</sub></i>

2



Dấu đẳng thức xảy ra <=> 1 2


1 2


.... <i>n</i>
<i>n</i>


<i>a</i> <i>a</i> <i>a</i>


<i>x</i> <i>x</i>  <i>x</i>


o Bất đẳng thức Trê- b-sép:


NÕu













<i>C</i>
<i>B</i>
<i>A</i>


<i>c</i>
<i>b</i>
<i>a</i>




3
.
3
3


<i>C</i>
<i>B</i>
<i>A</i>
<i>c</i>
<i>b</i>
<i>a</i>


<i>cC</i>
<i>bB</i>


<i>aA</i>    






NÕu












<i>C</i>
<i>B</i>
<i>A</i>


<i>c</i>
<i>b</i>
<i>a</i>





3
.
3
3


<i>C</i>
<i>B</i>
<i>A</i>
<i>c</i>
<i>b</i>
<i>a</i>
<i>cC</i>
<i>bB</i>


<i>aA</i>    






DÊu b»ng x¶y ra khi













<i>C</i>
<i>B</i>
<i>A</i>


<i>c</i>
<i>b</i>
<i>a</i>


<b>II - Một số bất đẳng thức phụ đã đợc chứng minh là đúng.</b>


o <i>x</i>2 <i>y</i>2 2<i>xy</i>



o <i>x</i>2<i>y</i>2 <i>xy</i> dÊu( = ) khi x = y = 0


o <i>x</i> <i>y</i>2 4<i>xy</i>



o  2


</div>
<span class='text_page_counter'>(2)</span><div class='page_container' data-page=2>

o


2


1 1 4



( , 0)


1


2 ( 0)


1 4


( , 0)


( )


<i>Khi b c</i>
<i>b c b c</i>


<i>b</i> <i>khi x</i>
<i>b</i>


<i>Khi x y</i>
<i>bc</i> <i>b c</i>


  




  


 





<b>III </b>–<b> Các bất đẳng thức trong tam giác</b>
<b>IV </b>–<b> Các hàm lợng giác thông dụng</b>
<b>V </b>–<b> Các tính chất cơ bản</b>


TÝnh chÊt 1: a > b <=> b < a


TÝnh chÊt 2: a > b vµ b > c => a > c
TÝnh chÊt 3: a > b <=> a + c > b + c
HÖ qu¶ : a > b <=> a - c > b – c


a + c > b <=> a > b – c


TÝnh chÊt 4 : a > c vµ b > d => a + c > b + d
a > b vµ c < d => a - c > b – d


TÝnh chÊt 5 : a > b vµ c > 0 => ac > bd
a > b vµ c < 0 => ac < bd


TÝnh chÊt 6 : a > b > 0 ; c > d > 0 => ac > bd
a > b > 0 => an<sub> > b</sub>n


a > b <=> an<sub> > b</sub>n<sub> víi n lỴ . </sub>


<b>VI </b>–<b> Các hằng đẳng thức đáng nhớ</b>
<b>VII </b>–<b> Các kiến thức về toạ độ vec tơ</b>


<b>VIII </b>–<b> C¸c kiÕn thøc vỊ tÝnh chÊt cđa tØ lƯ thøc:</b>





, ,


, , ,


<i>a</i> <i>a</i>


<i>a b c R</i>
<i>a b</i> <i>a b c</i>


<i>a</i> <i>c</i> <i>a</i> <i>a c</i> <i>c</i>


<i>a b c d R</i>


<i>b</i> <i>d</i> <i>b</i> <i>b d</i> <i>d</i>






  


  




     







<b>Phần II Các ph</b>– <b>ơng pháp chứng minh Bất đẳng thức</b>


Các phơng pháp chứng minh Bất đẳng thức vô cùng đa dạng ở đây tôi xin
trình bày những dạng phơng pháp thơng dụng nhất nh sau:


</div>
<span class='text_page_counter'>(3)</span><div class='page_container' data-page=3>

<i>Dạng 1 </i>–<i> Dựa vào định nghĩa và các phép biến đổi tơng đơng</i>


<i>Dạng 2 </i>–<i> Sử dụng bất đẳng thức Bunhiacopxky và các bất đẳng thức phụ.</i>
<i>Dạng 3 </i>–<i> Sử dụng Bất đẳng thức Cauchy</i>


<i>D¹ng 4 </i>–<i> Chøng minh bằng phản chứng</i>
<i>Dạng 5 </i><i> Phơng pháp lợng giác</i>


<i>Dạng 6 </i><i> Phơng pháp chứng minh qui nạp</i>


<i>Dạng 7 </i><i> Phơng pháp áp dụng các tính chất của các dÃy tỉ số bằng nhau</i>
<i>Dạng 8 </i><i> Phơng pháp dùng tam thức bậc hai</i>


<i>Dạng 9 </i><i> Phơng pháp dùng tính chất bắc cầu</i>


<i>Dng 10 - Phng phỏp dựng cỏc bt ng thức trong tam giác</i>
<i>Dạng 11 </i>–<i>Phơng pháp đổi biến số</i>


</div>
<span class='text_page_counter'>(4)</span><div class='page_container' data-page=4>

<b>Dạng 1- Dựa vào định nghĩa và các phép biến đổi tơng tơng đơng</b>


<i>Đây là phơng pháp cơ bản nhất, dựa vào các tính chất cơ bản của bất đẳng </i>
<i>thức đơn giản để biến đổi các bất đẳng thức phức tạp của đề ra thành các bất đẳng</i>


<i>thức đơn giản và đúng hoặc các bất đẳng thức đã đợc chứng minh là đúng. ở phần </i>
<i>này các bạn chú ý đến các hằng đẳng thức:</i>


 <i><sub>a</sub></i>2<sub></sub><sub>2</sub><i><sub>ab b</sub></i><sub></sub> 2 <sub></sub><sub>(</sub><i><sub>a b</sub></i><sub></sub> <sub>)</sub>2 <sub></sub><sub>0</sub>


 <i>a</i>2 <i>b</i>2 <i>c</i>2 2<i>ab</i>2<i>ac</i>2<i>bc</i> (<i>a b c</i> )2 0
Ph


ơng pháp:


<i>Khi biến đổi tơng đơng ta cố gắng làm xuất hiện các điều kiện đã cho trong </i>
<i>giả thiết nhằm áp dụng đợc điều kiện của giả thiết để chứng minh đợc bất </i>
<i>đẳng thức đó là đúng.</i>


 <i>Chuyển vế để chứng minh bất đẳng thức đó (</i>0; 0; 0; 0 )  


 <i>Chuyển vế các thừa số về dạng hằng đẳng thức để dể chứng minh</i>


 <i>Làm xuất hiện các tích các thừa số có chứa các yếu tố của đề bài để ta xét </i>
<i>dấu các thừa số đó</i>


 <i>Chia nhỏ từng vế để chứng minh sau đó cộng vế theo vế các bất đẳng thức </i>
<i>con để đợc điều phải chứng minh.</i>


<b>Mét sè vÝ dô:</b>


VÝ dô 1:


Chøng tá r»ng víi <i>a b</i>, 0 th×:



(<i>ax by bx ay</i><sub></sub> )( <sub></sub> ) (<sub></sub> <i>a b xy</i><sub></sub> )2 (1)


<b>Gi¶i</b>


2 2 2 2 2 2


2 2


2


(1) 2


( 2 ) 0


( ) 0


<i>abx</i> <i>a xy b yx bay</i> <i>a xy</i> <i>abxy b xy</i>


<i>ab x</i> <i>y</i> <i>xy</i>


<i>ab x y</i>


      


   


  


Bất đẳng thức ln đúng vì <i>a b</i>, 0.
Ví dụ 2:



Cho

<i>0 a b c</i>

<sub>  </sub>

Chøng minh r»ng:


</div>
<span class='text_page_counter'>(5)</span><div class='page_container' data-page=5>

<i>a b</i> <i>c</i> <i>b</i> <i>c</i> <i>a</i>
<i>b</i> <i>c</i>  <i>a</i> <i>a b</i>  <i>c</i>


<b>Gi¶i</b>


<i>a b</i> <i>c</i> <i>b</i> <i>c</i> <i>a</i>


<i>b</i>  <i>c</i> <i>a</i>  <i>a b</i>  <i>c</i>


2 2 2 2 2 2


1


(<i>a c b a c b b c c a a b</i>)


<i>abc</i>


     


1 (<i>a c b c</i>2 2 ) (<i>b a a b</i>2 2 ) (<i>c b c a</i>2 2 )


<i>abc</i> 


 <sub></sub>      <sub></sub>


2 2 2



2


1


( ) ( ) ( )


1


( )( )


1


( )( )( ) 0


<i>c a</i> <i>b</i> <i>ab b a</i> <i>c b a</i>
<i>abc</i>


<i>b a ca cb ab c</i>
<i>abc</i>


<i>b a c b c a</i>
<i>abc</i>


 


 <sub></sub>      <sub></sub>


    


    



<i>0 a b c</i>

<sub>  </sub>

.


VËy <i>a b</i> <i>c</i> <i>b</i> <i>c</i> <i>a</i>


<i>b</i> <i>c</i>  <i>a</i>  <i>a b</i>  <i>c</i>


VÝ dơ 3:


Víi <i>a b c </i>, , 0 chøng minh:
1 1 1


2( )


<i>a</i> <i>b</i> <i>c</i>


<i>bc ca</i>  <i>ab</i>  <i>a b c</i> 


<b>Gi¶i</b>


1 1 1


2( )


<i>a</i> <i>b</i> <i>c</i>


<i>bc ca</i>  <i>ab</i>  <i>a b c</i> 


2 2 2



2 2 2


2( ) ( 0)


2 2 2 0


<i>a</i> <i>b</i> <i>c</i> <i>bc ac ba</i> <i>do abc</i>


<i>a</i> <i>b</i> <i>c</i> <i>bc</i> <i>ac</i> <i>ab</i>


      


      


2
(<i>a b c</i>) 0


    Hiển nhiên đúng.


VËy <i>a</i> <i>b</i> <i>c</i> 2(1 1 1)


<i>bc ca</i> <i>ab</i>  <i>a b c</i>  .


VÝ dô 4: Chøng minh r»ng mäi a,b,c,d th× :


</div>
<span class='text_page_counter'>(6)</span><div class='page_container' data-page=6>

<b>Gi¶i</b>




2 2 2 2



2 2 2 2


2 2 2 2


(1) 1 ( ) 0


( ) ( ) ( ) 1 0


1 1 1 1


( ) ( ) ( ) ( ) 0


2 2 2 2


         


         


        


<i>a</i> <i>b</i> <i>c</i> <i>d</i> <i>a b c d</i>


<i>a</i> <i>a</i> <i>b</i> <i>b</i> <i>c</i> <i>c</i> <i>d</i> <i>d</i>


<i>a</i> <i>b</i> <i>c</i> <i>d</i>


VËy :

<i>a</i>

2

<sub></sub>

<i>b</i>

2

<sub></sub>

<i>c</i>

2

<sub></sub>

<i>d</i>

2

<sub>    </sub>

1

<i>a b c d</i>



<b>VÝ dô 5: Chøng minh r»ng nÕu: </b>

<i>a b</i>

<sub> </sub>

2

thì

<i><sub>a</sub></i>

3

<sub></sub>

<i><sub>b</sub></i>

3

<sub></sub>

<i><sub>a</sub></i>

4

<sub></sub>

<i><sub>b</sub></i>

4(1)


<b>Giải</b>




4 4 3 3


3 3


(1) 0


( 1) ( 1) 0


<i>a</i> <i>b</i> <i>a</i> <i>b</i>


<i>a a</i> <i>b b</i>


    


    


3 3


3 3


2 2 2 2


( 1) ( 1) ( 1) ( 1) ( 1) ( 1) 0


( 1)( 1) ( 1)( 1) 2 0



( 1) ( 1) ( 1) ( 1) 2 0


<i>a a</i> <i>b b</i> <i>a</i> <i>b</i> <i>a</i> <i>b</i>


<i>a</i> <i>a</i> <i>b</i> <i>b</i> <i>a b</i>


<i>a</i> <i>a</i> <i>a</i> <i>b</i> <i>b</i> <i>b</i> <i>a b</i>


            


         


       


Suy ra điều phải chứng minh.
Vì:




2 2 2


2 2 2


( 1) 0 ( 1) ( 1) 0


( 1) 0 ( 1) ( 1)


2 2 0



<i>a</i> <i>a</i> <i>a</i> <i>a</i>


<i>b</i> <i>b</i> <i>b</i> <i>b</i>


<i>a b</i> <i>a b</i>


      


     


 


<b>Bài tập áp dung:</b>


Bài 1: Cho a + b = 2. Chøng minh r»ng:

<i><sub>a</sub></i>

4

<sub></sub>

<i><sub>b</sub></i>

4

<sub></sub>

<sub>2</sub>


Bµi 2:Chứng minh rằng với mọi số nguyên dơng n ta cã:


1 1 ... 1 2
2 3 2  (<i>n</i>1) <i>n</i> 


Bài 3: Chứng minh m,n,p,q ta đều có


m2 <sub>+ n</sub>2 <sub>+ p</sub>2 <sub>+ q</sub>2 <sub>+1 m(n + p + q +1) </sub>


Bµi 4: Chøng minh r»ng:

(a

10

<sub></sub>

b )(a

10 2

<sub></sub>

b ) (a

2

<sub></sub>

8

<sub></sub>

b )(a

8 4

<sub></sub>

b )

4


</div>
<span class='text_page_counter'>(7)</span><div class='page_container' data-page=7>

Bài 5: Chứng minh bất đẳng thức :


3
3



3


2


2 





 

<i>b</i> <i>a</i> <i>b</i>


<i>a</i> <sub> Trong đó : a > 0 , b > 0</sub>


Bµi 6: Chøng minh r»ng: Víi mäi sè d¬ng a, b, c, d ta cã:


<i><b> </b></i>


2
d
c
b
a
a
d


d
d


c


c
c
b


b
b
a


a


2
2


3
2
2


3
2
2


3
2
2


3















<b>Dạng 2 – Sử dụng bất đẳng thức Bunhiacôpsky và các bất đẳng thức phụ</b>


<i>Đây là phơng pháp phổ biến nhất trong việc chứng minh Bất đẳng thức. Chúng ta </i>
<i>dựa vào điều kiện đã cho ở đề bài để ta lựa chọn phơng pháp cho thích hợp. Ngồi </i>
<i>ra, ta cần phải chú ý đến dấu của BĐT để có thể sử dụng bất đẳng thức nào để </i>
<i>chứng minh. Khi áp dụng các BĐT đã đợc chứng minh là đúng thì bạn nên tách </i>
<i>nhỏ BĐT cần chứng minh ra thành các vế nhỏ sau đó cộng vế theo vế để đợc BĐT </i>
<i>cần chứng minh.</i>


<b>Mét sè vÝ dô:</b>


VÝ dô 1:


Chøng minh r»ng víi mäi sè thùc d¬ng x,y,z ta cã:




2 2 2



2 2 2


(

3

3



(

)(

)

9



<i>xyz x y z</i>

<i>x</i>

<i>y</i>

<i>z</i>


<i>x</i>

<i>y</i>

<i>z</i>

<i>xy yz zx</i>



  







<b>Gi¶i</b>




2 2 2 2


2 2 2


2 2 2 <sub>3</sub> 2


2
3


3(

) (

)




3(



3



3



<i>x</i>

<i>y</i>

<i>z</i>

<i>x y z</i>



<i>x y z</i>

<i>x</i>

<i>y</i>

<i>z</i>



<i>x</i>

<i>y</i>

<i>z</i>

<i>xyz</i>



<i>xy yz zx</i>

<i>xyz</i>



 



  







Do đó ta có:




2 2 2 2 2 2


2 2 2 <sub>2</sub> <sub>2</sub> <sub>2</sub> <sub>3</sub> <sub>2</sub>



3
3


(

)

(( 3 1)

)



(

)(

)

<sub>(</sub>

<sub>)(3</sub>



3 1

3 1 1

3

3



3

3

3

9



<i>xyz x y z</i>

<i>x</i>

<i>y</i>

<i>z</i>

<i>xyz</i>

<i>x</i>

<i>y</i>

<i>z</i>



<i>x</i>

<i>y</i>

<i>z</i>

<i>xy yz zx</i>

<i><sub>x</sub></i>

<i><sub>y</sub></i>

<i><sub>z</sub></i>

<i><sub>xyz</sub></i>



<i>xyz</i>


<i>xyz</i>



  





<sub></sub>

<sub></sub>





</div>
<span class='text_page_counter'>(8)</span><div class='page_container' data-page=8>

DÊu “=” x¶y ra khi x=y=z


VÝ dơ 2: Chøng minh r»ng: 19942000 <sub></sub>19952000 <sub></sub>19962000 (1)



<b>Gi¶i</b>


(1) (1994)2000 1 (1996)2000 (1 1 )2000


1995 1995 1995


    


Theo bất đẳng thức Becnuli ta có:


(1 1 )2000 1 2000 1 (1994)2000


1995  1995   1995


V×: 2000 1 (1994)2000
1995  1995
VÝ dô 3:


Cho <sub>a b 2</sub><sub></sub> <sub></sub> Chøng minh r»ng: <sub>a</sub>4<sub></sub><sub>b</sub>4 <sub></sub><sub>2</sub>


<b>Gi¶i</b>


áp dụng bất đẳng thức Bunhiacopxki cho 4số 1,1,a,b ta có:




2 2 2 2 2


2 2 2



2 2
2 2


(1.a 1.b) (1 1 )(a b )


(a b) 2(a b )


4 2(a b )


2 a b


   


   


  


  


áp dụng bất đẳng thức Bunhiacopxki cho 4số 1,1,a2<sub>,b</sub>2<sub> ta có:</sub>




   


    


  


  



2 2 2 2 4 4


2 2 4 4


4 4
4 4


(1.a 1.b ) (1 1 )(a b )


2 (a b ) 2(a b )


4 2(a b )


a b 2


VÝ dô 4: Cho a,b,c>0 Chøng minh r»ng:

1 1 1

9


a b c

a b c



<b>Gi¶i</b>


Ta cã:


</div>
<span class='text_page_counter'>(9)</span><div class='page_container' data-page=9>



1 1 1 a a b b c c


(a b c)( ) 1 1 1


a b c b c a c a b



a b c a b c


3 ( ) ( ) ( ) 9


b a a c c b


            


       


V× : a b 2
b a 




c a
2
a c


b c


2
c b


 


 


Nªn: 3 (a b) (c a) (b c) 9



b a a c c b


      


VÝ dô 5: Cho 4 sè d¬ng a,b,c,d chøng minh r»ng:


a b c d 2
b c c d a d a b       


<b>Gi¶i</b>


áp dụng bất đẳng thức phụ:


1 1 <sub>2</sub> (x,y>0)
xy (x y)


Ta cã:




2 2


2


a c a(d a) c(b c) a c ad bc


4


b c d a (b c)(d a) (a b c d)



     


  


      


T¬ng tù:




2 2


2


b d b d ab cd


4


c d a b (a b c d)


  


 


    


Céng vÕ theo vÕ ta cã:





2 2 2 2


2


a b c d a b c d ad bc ab cd


4


b c c d a d a b (a b c d)


      


   


      


</div>
<span class='text_page_counter'>(10)</span><div class='page_container' data-page=10>



2 2 2 2


2


2 2 2 2 2


2 2 2 2


2 2


a b c d ad bc ab cd



4 2


(a b c d)


4a b c d ad bc ab cd 2(a b c d)


2a 2b 2c 2d 4ac 4bd 0


(a c) (b d) 0


      




  


           


      


   


<b>Bài tập áp dụng:</b>


Bài 1: Cho x,y,z thoà m·n x(x 1) y(y 1) z(z 1) 4
3


     



Chøng minh r»ng: x y z 4  


Bµi 2: Cho a>b>c>0 vµ 2 2 2 1


<i>b</i> <i>c</i>


<i>a</i> .Chøng minh r»ng




3 3 3 <sub>1</sub>


2


<i>a</i> <i>b</i> <i>c</i>


<i>b c a c a b</i>     


Bµi 3: Cho x , y lµ 2 sè thùc tho¶ m·n x2<sub> + y</sub>2<sub> = </sub><i><sub>x</sub></i> <sub>1</sub> <i><sub>y</sub></i>2 <i><sub>y</sub></i> <sub>1</sub> <i><sub>x</sub></i>2





Chøng minh r»ng : 3x + 4y  5


Bµi 4: Cho a, b, c  0 ; a + b + c = 1 . Chøng minh r»ng:


6







<i>b</i> <i>b</i> <i>c</i> <i>c</i> <i>a</i>


<i>a</i>


Bài 5:Cho a, b, c là độ dài 3 cạnh một tam giác, p là nửa chu vi.
Chứng minh rằng:


3

(1)



<i>p</i>

<i>p</i>

<i>a</i>

<i>p</i>

<i>b</i>

<i>p</i>

<i>c</i>

<i>p</i>



Bµi 6: Cho a, b,c là 3 số khác 0. Chứng minh rằng:




2 2 2


2 2 2


<i>a</i> <i>b</i> <i>c</i> <i>a b</i> <i>c</i>


<i>b</i> <i>c</i> <i>a</i>  <i>b</i> <i>c</i> <i>a</i>


Bài 7 Cho ba số <i>a</i>,<i>b</i>,<i>c</i>0.Thoả mÃn abbccaabc



Chøng minh r»ng:


<i><b> </b></i> 2 2 2 3


2
2
2
2
2
2









<i>ca</i>
<i>c</i>
<i>a</i>
<i>bc</i>


<i>b</i>
<i>c</i>
<i>ab</i>


<i>a</i>


<i>b</i> <i><b><sub> </sub></b></i><sub>(*)</sub>



<b>Dạng 3 – sử dụng bất đẳng thức Cauchy</b>


</div>
<span class='text_page_counter'>(11)</span><div class='page_container' data-page=11>

<i>Đây là phơng pháp chứng minh BĐT mà học sinh THCS dễ nhận dạng để chứng </i>
<i>minh đó là sử dụng Bất đẳng thức Cauchy . Ta cần phải chú ý đến dấu của BĐT để </i>
<i>có thể sử dụng bất đẳng thức nào để chứng minh. Khi áp dụng các BĐT đã đợc </i>
<i>chứng minh là đúng thì bạn nên tách nhỏ BĐT cần chứng minh ra thành các vế nhỏ</i>
<i>sau đó cộng vế theo vế để đợc BĐT cần chứng minh.</i>


VÝ dô 1: Cho 3 sè d¬ng a,b,c chøng minh r»ng:




3 3 3


3 3 3


a

b

c

a b c



b

c

a

b c a



<b>Gi¶i</b>


Vận dụng bất đẳng thức Cơsi, ta có:




  


  



3 3


3 3


3 3


3 3


a a a


1 3 (1)
b


b b


b b b


1 3 (2)
c


c c


3 <sub></sub> 3 <sub> </sub>


3 3


c c c


1 3 (3)


a


a a


Céng vÕ theo vÕ (1) (2) vµ (3) ta cã:




3 3 3


3 3 3


a

b

c

a b c

a b c



2(

) 3 2(

)



b

c

a

b

c a

b

c a



a b c



2(

) 3



b

c a







VËy:



3 3 3


3 3 3


a b c a b c


b  c  a b c a 


VÝ dơ 2: Cho a,b,c >0 tho¶ m·n

1

1

1

2


1 a

1 b

1 c



Chøng minh r»ng: abc 1
8


</div>
<span class='text_page_counter'>(12)</span><div class='page_container' data-page=12>

Ta cã:

1

1

1

1

1

b

c



1 a

 

1 b

 

1 c

1 b 1 c



áp dụng bất đẳng thức Côsi:






  




  



1 bc


2


1 a (1 b)(1 c)


1 ac


2


1 a (1 a)(1 c)




  


1 ab


2


1 c (1 a)(1 b)


Nhân lại ta đợc: 1 8abc


(1 a)(1 b)(1 c)   (1 a)(1 b)(1 c)  


abc 1


8



 


VÝ dụ 3: Giả sử a,b,c d, là 4 số dơng tho· m·n:


1 1 1 1 3
1 a 1 b 1 c 1 d       


Chøng minh r»ng: abcd 1
81


<b>Gi¶i</b>


Tõ gi¶ thiÕt ta cã:




        


   


    


   


1 <sub>1</sub> 1 <sub>1</sub> 1 <sub>1</sub> 1 <sub>1 3 4</sub>


1 a 1 b 1 c 1 d



a b c d <sub>1</sub>


1 a 1 a 1 a 1 a




     


  


   


   


 


     


a(1 b) b(1 a) c(1 d) d(1 c)
1


(1 a)(1 b) (1 c)(1 d)


a b 2ab c d 2cd


1 a b ab 1 c d cd
áp dụng bất đẳng thức Cơsi ta có:


</div>
<span class='text_page_counter'>(13)</span><div class='page_container' data-page=13>

1 2 ab 2ab 2 cd 2cd 2 ab 2 cd



1 2 ab ab 1 2 cd cd 1 ab 1 cd


 


   


     




4


4 4


4 <sub>4</sub> <sub>2</sub>


4 4


4


abcd abcd


1 2 2 4


1 ab cd abcd 1 ab cd abcd


4 abcd 4 abcd


1



1 2 abcd abcd <sub>(1</sub> <sub>abcd)</sub>


1 abcd 4 abcd


1 3 abcd
1
abcd


8


 


    


     


 


 


  


  <sub></sub>


  







<b>Bài tập áp dụng:</b>


Bài 1: Chứng minh rằng: ( a+ b + c ) (


<i>a</i>


1


+


<i>b</i>


1


+


<i>c</i>


1


) ≥ 9 víi a,b,c > 0


Bài 2: Cho a,b,c là độ dài 3 cạnh của một tam giác với chu vi 2p
Chứng minh rằng:


a)(p a)(p b)(p c) abc
8


   



b) 1 1 1 2(1 1 1)


p a p b p c      a b c 


Bµi 3: Cho a, b, c  0 ; a + b + c = 1 . Chøng minh r»ng:
<i>a</i>1 <i>b</i>1 <i>c</i>13,5


Bài 4:Cho a, b, c là độ dài 3 cạnh và 2p là chu vi của một tam giác.
Chứng minh rằng:


( )( )( )
8


<i>abc</i>
<i>p a p b p c</i> 


<b>Dạng 4 </b><b> Chứng minh bằng phản chứng</b>


</div>
<span class='text_page_counter'>(14)</span><div class='page_container' data-page=14>

<i>A B</i> <i> đúng, từ đó chứng minh những lập luận chính xác ta suy ra điều mâu thuẩn </i>


<i>từ giả thiết. Kết luận <sub>A B</sub></i><sub></sub> <i> đúng. Điều vơ lý có thể là trái với giả thiết, hoặc là </i>
<i>những điều trái ngợc nhau , từ đó suy ra đẳng thức cần chứng minh là đúng.</i>
<i>Một số hình thức chứng minh bằng phản chứng:</i>


<i> Dùng mệnh đề đảo.</i>


<i> Phủ định rồi suy ra điều trái với giả thiết.</i>
<i> Phủ định rồi suy ra trái với điều đúng.</i>
<i> Phủ định rồi suy ra hai điều trái ngợc nhau.</i>



<b>Mét sè vÝ dơ:</b>


VÝ dơ 1: Cho a,b,c,d R vµ <sub>a b 2cd</sub><sub></sub> <sub></sub>


Chứng minh rằng ít nhất một trong hai bất đẳng thức sau đây là đúng
<sub>c</sub>2 <sub></sub><sub>a,d</sub>2 <sub></sub><sub>b</sub>


<b>Gi¶i</b>


Giả sử hai bất đẳng thức trên đều sai, có nghĩa ta đợc :
<sub>c</sub>2 <sub></sub> <sub>a</sub><sub> và </sub><sub>d</sub>2 <sub></sub> <sub>b</sub>


<sub></sub> <sub>c</sub>2<sub></sub> <sub>a 0</sub><sub></sub> vµ <sub>d</sub>2<sub></sub> <sub>b 0</sub><sub></sub>




2 2


2 2
2 2


c a d b 0


c d (a b) 0


c d 2cd 0


    


    



   


V× a+b =2cd


 (c d) 2  0 M©u thuÉn


Nên sẽ có ít nhất một trong hai bất đẳng thức đã cho là đúng


Ví dụ 2: Cho 3 số dơng a,b,c nhỏ hơn 2. Chứng minh rằng có ít nhất một trong các
bất đẳng thức sau là sai:




a(2 a) 1
b(2 b) 1
c(2 c) 1


 


 


 


<b>Gi¶i</b>


Giả sử các bất đẳng thức sau đều đúng, nhân ba đẳng thức lại ta đợc
a(2 a)b(2 c)c(2 c) 1   


</div>
<span class='text_page_counter'>(15)</span><div class='page_container' data-page=15>

Mµ 0 a(2 a) 2a a    2  1 (a 1) 2 1



T¬ng tù ta cã:


0 b(2 b) 1


0 c(2 c) 1


  


  


Suy ra:


abc(2 a)(2 b)(2 c) 1   


M©u thn


Vậy có ít nhất một trong các bất đẳng thức đã cho là sai


Ví dụ 3: Cho 6 số tự nhiên khác 0 nhỏ hơn 108. Chứng minh rằng có thể chọn đợc
3 trong 6 số đó, chẳng hạn a,b,c sao cho a<bc, b<ca, c<ab


<b>Giải</b>


Giả sử 6 số tự nhiên khác 0 là 1 a <sub>1</sub> a<sub>2</sub> ... a <sub>6</sub> 108


Râ rµng a<sub>2</sub> 2; a<sub>3</sub> 3 Víi 3 sè x,y,z tho· m·n 1 x y z


Ta luôn có x<yz và y<xz. Nếu trong các số a1, a2 ,, a6 không có 3 số nào thoÃ



mÃn a<b<c và c<ab thì cã

a

<sub>4</sub>

a a

<sub>2 3</sub>

6,



5 4 3


6 5 4


a

a a

6.3 18



a

a a

18.6 108







Trái với giả thiết a6 <108. VËy ph¶i cã 3 sè a,b,c tho· m·n a<bc; b<ca; c<ab


VÝ dơ 4: Cho c¸c sè thùc a,b,c tho· m·n ®iỊu kiƯn:




a b c 0 (1)
ab+bc+ca>0 (2)
abc>0 (3)
   










Chøng minh r»ng: a,b,c >0


<b>Gi¶i</b>


Giả sử trong 3 số thực a,b,c đã cho có một số âm hay bằng 0, giả sử số đó là a 0


</div>
<span class='text_page_counter'>(16)</span><div class='page_container' data-page=16>



a 0 a 0


a 0
abc 0


b>0 b<0


a 0 bc 0


c<0 c>0


 <sub></sub>  <sub></sub>




 <sub></sub> <sub></sub>


 


   



  


 <sub></sub>  <sub></sub>  


   


 


 




Xét khả năng a 0; b>0; c<0 a+c<0


Ta cã:




2


2 2 2


(1) : a b c 0 b>-(a+c) (a+c)b<-(a+c)


(a c)b ca (a c) ac (a ac c )


ab bc ca 0


    



         


   


V× : (a2ac c 2  0 a,b,c R) 


Điều này mâu thuẫn với giả thiết.
Vậy 3 sụ a,b,c u l s dng.


<b>Bài tập áp dụng:</b>


Bài 1:


Cho 0<i>a b c</i>, ,  .Chứng minh rằng ít nhất có một bất đẳng thức sau đây là sai:1


(1 ) 1 ; (1 ) 1 ; (1 ) 1


4 4 4


<i>a</i>  <i>b</i>  <i>b</i>  <i>c</i>  <i>c</i>  <i>a</i> 


Kết quả này mâu thuẩn với kết quả của giả thiết đã nêu ra ở trên.
Vậy ít nhất phải có một bt ng thc sai.


Bài 2:


Cho 25 số tự nhiên

<i>a a</i>

1

, ,...,

2

<i>a</i>

25 thoả mản điều kiện





1 2 25


1

1

<sub>...</sub>

1

<sub>9</sub>



<i>a</i>

<i>a</i>

 

<i>a</i>

.


Chứng minh rằng trong 25 số tự nhiên đó, tồn tại hai số bằng nhau.


<b>D¹ng 5 </b><b> Phơng pháp lợng giác</b>


<i>õy l mt trng hp đặc biệt của phơng pháp đổi biến số. Đối với học sinh THCS </i>
<i>thì việc sử dụng phơng pháp này là khá mới vì kiến thức cơ bản của phần lợng giác </i>


</div>
<span class='text_page_counter'>(17)</span><div class='page_container' data-page=17>

<i>cha đợc nghiên cứu sâu. Cho nên ở phơng pháp này tơi xin trình bày một số kiến </i>
<i>thức lý thuyết và các dạng phơng pháp một cách chi tiết hơn. </i>


<b>KiÕn thøc cÇn nhí:</b>


<i><b>1. Các hệ thức cơ bản</b></i>


+ cos2 sin2 1




+ 1 + tg2 = ( <sub>2</sub> k )


cos
1



2  







+ tg . cotg = 1 ( 


2
k


) + 1 + cotg2<sub> = </sub> ( k )


sin
1


2  




2. Công thức cộng, công thức hạ bậc, công thức nhân đôi, cơng thức biến tích thành
tổng và cơng thức biến tổng thành tích. Chúng ta dựa vào các trơng hợp dới đây để
có thể đổi biến lợng giác một cách chính xỏc.


Một số phơng pháp lợng giác thờng gặp:


Nu thy x2<sub> + y</sub>2<sub> = 1 thì đặt </sub>












cos


y



sin


x



víi   [0, 2]


 Nếu thấy x2<sub> + y</sub>2<sub> = a</sub>2<sub> (a > 0) thì đặt </sub>











cos


a



y



sin


a


x



víi   [0, 2]


 Nếu thấy |x|  1 thì đặt




sin ;


2 2


cos 0;


<i>x</i> <i>khi</i>


<i>x</i> <i>khi</i>


 







  



  


 <sub></sub> <sub></sub>




  




 Nếu thấy |x|  m ( <i>m </i>0) thì đặt




sin ;


2 2


cos 0;


<i>x m</i> <i>khi</i>


<i>x m</i> <i>khi</i>


 


 


  



  


  


  


 




  




 Sư dơng c«ng thøc: 1+tg2 = 1<sub>2</sub> 2 1<sub>2</sub> 1 ( )


cos <i>tg</i> cos 2 <i>k</i>




   


      


 Nếu |x| 1 hoặc bài toán có chứa biểu thøc x2 1


thì đặt x =




cos


1


víi  







 








 


2
3
,
2


</div>
<span class='text_page_counter'>(18)</span><div class='page_container' data-page=18>

 NÕu |x|  m hoặc bài toán có chứa biểu thức 2 2


m
x 



thì đặt x =



cos


m


víi  







 








 


2
3
,
2


;
0



 Sư dơng c«ng thøc 1+ tg2<sub> = </sub>



2
cos


1


.


 Nếu x  R và bài toán chứa (1+x2<sub>) thì đặt x = tg với   </sub> <sub></sub>








  


2
,
2


 Nếu x  R và bài tốn chứa (x2<sub>+m</sub>2<sub>) thì đặt x = mtg với   </sub> <sub></sub>









  


2
,
2


VÝ dô 1: Cho a,b,c,d R Víi <sub>a c 1 d</sub><sub></sub> <sub></sub> 2 Vµ <sub>b d 1 c</sub><sub></sub> <sub></sub> 2
Chøng minh r»ng a  b 1


<b>Gi¶i</b>


Víi: <sub>a c 1 d</sub><sub></sub> <sub></sub> 2Vµ<sub>b d 1 c</sub><sub></sub> <sub></sub> 2 Ta cã:




2 2


2 2


1 d 0 d 1 -1 d 1


-1 c 1


1 c 0 c 1



     <sub></sub>  


  




  


 


   


  


 


Do đó ta đặt: d cosb và c cosa với , 0;
2
p
a b  


 


 


<sub></sub> <sub>a</sub> <sub></sub><sub>c 1 d</sub><sub></sub> 2 <sub></sub><sub>cos</sub><sub>a</sub> <sub>1 cos</sub><sub></sub> 2<sub>b</sub><sub></sub><sub>cos sin</sub><sub>a</sub> <sub>b</sub>


Vµ <sub>b</sub> <sub></sub><sub>d 1 c</sub><sub></sub> 2 <sub></sub><sub>cos 1 cos</sub><sub>b</sub> <sub></sub> 2<sub>a</sub> <sub></sub><sub>cos sin</sub><sub>b</sub> <sub>a</sub>


a b cos sin cos sin



sin( ) 1


a b b a


b a


   


  


VËy: a  b 1


VÝ dô 2: Chøng minh r»ng:




2


2


(1 x )sina 2xcosa


1 x,a R
1 x









<b>Giải</b>


Đặt x tg sin
cos


a
a


a


Với ;


2 2
p p
a <sub></sub> <sub></sub>


  Th×


</div>
<span class='text_page_counter'>(19)</span><div class='page_container' data-page=19>



2


2 <sub>2</sub>


2
2


2



2 2


2 2


sin sin


(1 )sina 2 cosa


(1 x )sina 2xcosa <sub>cos</sub> <sub>cos</sub>


sin
1 x


(1 )


cos


(cos sin )sina 2sin cos cosa


cos sin


cos2 sina sin2 cosa


a a


a a



a
a


a a a a


a a


a a


 


 







 






 


sin(a 2 ) 1 a 


Ví dụ 3: Chứng minh rằng nếu x 1 và n là số nguyên lớn hơn 1 thì ta có bất
đẳng thức:



(1 x) n (1 x) n  2n


<b>Gi¶i :</b>


Vì: x 1nên ta đặt x cost với






n n n n


2 n 2 n


n 2 n 2 n n


t ;


(1 x) (1 x) (1 cost) (1 cost)


t t


(2cos ) (2sin )


2 2


t t


2 (cos ) (sin ) 2 (1)



2 2


p p
 


       


 


 


  


 


 


Do


2 2 2 n


2 2 2 n


2 n 2 n


t t t


0 cos 1 cos (cos )



2 2 2


t t t


0 sin t sin (sin )


2 2 2


t t


1 (cos ) (sin )


2 2


 <sub></sub> <sub></sub>  <sub></sub>


 




 


    


 


  


(1)



 đúng


Vậy bất đẳng thức đã đợc chứng minh.
Ví dụ 4:


Chøng minh r»ng: 1 1 a2

(1a)3  (1 a)3

2 2 2 2a2 (1)


<b>Giải:</b>


</div>
<span class='text_page_counter'>(20)</span><div class='page_container' data-page=20>

Đặt a=cos với [0,]       ; 1 a sin
2
cos
2
a
1
;
2
sin
2
a
1 2
(1)
2
cos
2
sin
2
2
2
2


2
sin
2
cos
2
2
.
2
cos
2
sin
2


1 3 3 <sub></sub> <sub></sub>  







 






2
cos
2


sin
1
2
sin
2
cos
2
sin
2
cos
2
sin
2
cos
2
cos
2


sin 2 2 <sub></sub> <sub></sub>  







 











 







 



 cos 1


2
sin
2
cos
2
sin
2
cos
2
cos


2


sin 2 2














 







 



đúng  (đpcm)



<b>Bµi tËp ¸p dơng:</b>


Bµi 1:


Cho a2<sub> + b</sub>2<sub> - 2a - 4b + 4 = 0. Chøng minh r»ng:</sub>


A = a2<sub></sub> b2<sub></sub>2 3ab<sub></sub> 2(1<sub></sub>2 3)a<sub></sub>(4<sub></sub> 2 3)b<sub></sub>4 3<sub></sub> 3<sub></sub>2


Bµi 2: Cho a, b tho¶ m·n : 5a12b7= 13
Chøng minh r»ng: a2<sub> + b</sub>2<sub> + 2(b-a)  - 1</sub>


Bµi 3:


Chøng minh r»ng: 3 2A2 3a22a 1 a2  32


Bµi 4:


Chøng minh r»ng A =


2 <sub>1</sub> <sub>3</sub>


2 1
<i>a</i>
<i>a</i>
<i>a</i>
 
  
Bµi 5:


Chøng minh r»ng: - 4  A =



2
2
a
1
a
12


5  <sub> 9 </sub><sub></sub><i><sub>a</sub></i> <sub></sub><sub>1</sub>


Bµi 6:


Chøng minh r»ng: a,b,c


)
a
1
)(
c
1
(
|
a
c
|
)
c
1
)(
b


1
(
|
c
b
|
)
b
1
)(
a
1
(
|
b
a
|
2
2
2
2
2


2 <sub></sub> <sub></sub> 












Bµi 7:


Chøng minh r»ng: ab cd  (ac)(bd) (1) a,b,c,d0 (1)


Bµi 8:


</div>
<span class='text_page_counter'>(21)</span><div class='page_container' data-page=21>

Chøng minh r»ng: <sub>2</sub>1


)
b
1
)(
a
1
(


)
ab
1
)(
b
a
(


2



2 








 a, b  R


<b>D¹ng 6 </b>–<b> Phơng pháp chứng minh qui nạp</b>


<i>Phng phỏp qui np thng sử dụng để chứng minh một bất đẳng thức phụ thuộc </i>
<i>vào số nguyên dơng n. Ta thực hiện các bớc sau:</i>


 <i>Kiểm nghiệm để chứng tỏ BĐT đúng với điều kiện nhỏ nhất.</i>


 <i>Giả sử BĐT đúng với một số nguyên dơng k bất kỳ </i>


 <i>Cần chứng minh BĐT cũng đúng với n = k + 1</i>


<b>VÝ dô 1: Chøng minh r»ng: </b><sub>2</sub>n <sub></sub><sub>2n 1</sub><sub></sub> Víi mäi số dơng n 3


<b>Giải:</b>


Vi n=3 thỡ <sub>2</sub>3 <sub> </sub><sub>8 2.3 1 7</sub><sub></sub> <sub></sub> đúng


Giả sử bất đẳng thức đúng với n=k bất kì có nghĩa là:
2k 2k 1  2.k.2 (2k 1).2 



Ta cÇn chøng minh:
2k 1  2(k 1) 1 


Theo gt quy n¹p ta cã:


2k 1 (2k 1)2 4k 2 2k 2k 2 2(k 1) 1


Điều phải chứng minh.


<b>VÝ dơ 2: Chøng minh r»ng víi mäi sè tù nhiªn </b>n 2


Ta cã:


1 1 ... 1 13
n 1 n 2    2n  24


<b>Gi¶i:</b>


a. Víi n=2 ta cã:


1 1 13 14 13
3 4  24 24  24 đúng


</div>
<span class='text_page_counter'>(22)</span><div class='page_container' data-page=22>

(

<sub>1</sub>

<sub>2</sub>

) (

<sub>1</sub>

<sub>3</sub>

) ... (

<sub>1</sub>

)



(

<sub>2</sub>

<sub>3</sub>

) ... (

<sub>2</sub>

) ... (

<sub>1</sub>

)



1

1



(

<sub>1 2</sub>

<sub>2 1</sub>

)




1

1

1

1



(

<sub>1 3</sub>

<sub>3 1</sub>

) ... (

<sub>1</sub>

<sub>1</sub>

) ...



1

1



(

<sub>1</sub>

<sub>1</sub>

) (1)



<i>n</i>

<i>n</i>

<i>n</i>

<i>n</i>

<i>n</i>

<i>n</i>



<i>x</i>

<i>x</i>

<i>x</i>

<i>x</i>

<i>x</i>

<i><sub>xn</sub></i>



<i>n</i>

<i>n</i>

<i>n</i>

<i>n</i>

<i>n</i>

<i>n</i>



<i>x</i>

<i>x</i>

<i>x</i>

<i>x</i>

<i><sub>n</sub></i>

<i>x</i>

<i><sub>n</sub></i>

<i>x</i>

<i><sub>n</sub></i>



<i>n</i>

<i>n</i>



<i>x x</i>

<i>x x</i>



<i>n</i>

<i>n</i>

<i>n</i>

<i>n</i>



<i>x x</i>

<i>x x</i>

<i>x x</i>

<i><sub>n</sub></i>

<i>x x</i>

<i><sub>n</sub></i>



<i>n</i>

<i>n</i>



<i>x</i>

<i><sub>n</sub></i>

<i>x</i>

<i><sub>n</sub></i>

<i>x x</i>

<i><sub>n</sub></i>

<i><sub>n</sub></i>
























1 1 ... 1 13


k 2 k 3    2k 2 24 
Ta cã:


1 1 ... 1 ( 1 ... 1) 1 1 1


k 2 k 3    2k 2  k 1   2k 2k 1 2k 2 k 1    


V× :



1 ... 1 13
k 1  2k  24
Nªn:


13 1 1 1 13


24 2k 1 2k 2 k 1 24       đúng.


Ví dụ 3: Chứng minh bất đẳng thức Côsi trong trờng hợp tổng quát.
Với

<i>a</i>

<sub>1</sub>

, a ... a R , n 2

<sub>2</sub> <sub>n</sub>

<sub>n</sub>

thì


1 2


1 2

...

<i><sub>n</sub></i> <i><sub>n</sub></i>

<sub>. ...</sub>



<i>n</i>


<i>a a</i>

<i>a</i>

<i><sub>a a a</sub></i>



<i>n</i>



<sub></sub>



<b>Gi¶i:</b>


 Với

<i>n</i>

<i>=2 bất đẳng thức đả đợc chứng minh ở 1. (bất đẳng thức Ơclit)</i>


NÕu



1 2 1

<i>n</i>

1

2

<i>n</i>

1



<i>x x</i>

<i>x</i>

<i>x</i>

.

<i>x</i>

<sub>1 2</sub>

, x

<i>R</i>



VËy

<i>x</i>

<sub>1 2</sub>

, x

<i>R</i>

thì ta luôn có (chuyển một bộ phận sang vế phải, ta


đ-ợc)


1 2 1 2


1 2 1 2 2 1


1

1



(

)(

) 0



1

<sub>1.</sub>



<i>n</i>

<i>n</i>



<i>x</i>

<i>x</i>

<i>x x</i>



<i>n</i>

<i>n</i>

<i>n</i>

<i>n</i>



<i>x</i>

<i>x</i>

<i>x x</i>

<i>x x</i>



<sub></sub>

<sub></sub>

<sub></sub>








Lấy

<i>n</i>

số thực không âm

<i>x x</i>

<sub>1 2</sub>

, ...

<i>x</i>

<i><sub>n</sub></i>

<i>R</i>

,

viết các bất đẳng thức tơng ứng rồi


cộng lại ta đợc:


</div>
<span class='text_page_counter'>(23)</span><div class='page_container' data-page=23>

Từ đó:




1 1 1


( 1)( <sub>1</sub> <sub>2</sub> ... ) <sub>1 2</sub>( <sub>3</sub> ... )


1 1 1 1 1 1


( ... ) ( ... ) (2)


2 1 3 1 2 1


<i>n</i> <i>n</i> <i>n</i> <i>n</i> <i>n</i> <i>n</i>


<i>n</i> <i>x</i> <i>x</i> <i>x<sub>n</sub></i> <i>x x</i> <i>x</i> <i>x<sub>n</sub></i>


<i>n</i> <i>n</i> <i>n</i> <i>n</i> <i>n</i> <i>n</i>


<i>x x</i> <i>x</i> <i>x<sub>n</sub></i> <i>x x<sub>n</sub></i> <i>x</i> <i>x<sub>n</sub></i>


  



       


     


       






Bây giờ theo giả thiết quy nạp, ta thừa nhận rằng đối với

<i>n </i>

1

số thực không âm
bất kì , trung bình cộng khơng nhỏ hơn trung bình nhân của chúng. Thế thì nói
riêng ta có:




<i>x</i><sub>2</sub><i>n</i>1<i>x</i><sub>3</sub><i>n</i>1 ... <i>xnn</i>1

<sub></sub>

(<i>n</i>1)<i>x x</i><sub>2 3</sub>...<i>xn</i>




<i>x</i><sub>1</sub><i>n</i>1<i>x</i><sub>3</sub><i>n</i>1 ... <i>xnn</i>1

<sub></sub>

(<i>n</i>1)<i>x x</i><sub>1 3</sub>...<i>xn</i>


………


1 1 ... 1 ( 1) ...


1<i>n</i> 2<i>n</i> 1<i>n</i> 1 2 1



<i>x</i>  <i>x</i>   <i>x<sub>n</sub></i>  <i>n</i> <i>x x</i> <i>x<sub>n</sub></i>




Sử dụng các bất đẳng thức này, ta có thể tăng cờng các bất đẳng thức
( 2 )


(

<i>n</i>

1)(

<i>x</i>

<sub>1</sub>

<i>n</i>

<i>x</i>

<sub>2</sub>

<i>n</i>

 

...

<i>x</i>

<i><sub>n</sub></i>

<i>n</i>

)

<i>n n</i>

(

1)

<i>x x</i>

<sub>1 2</sub>

... )

<i>x</i>

<i><sub>n</sub></i>



Trong hệ thức này đặt <i>x</i><sub>1</sub><i>n</i> <i>a x</i><sub>1 2</sub>, <i>n</i> <i>a</i><sub>2</sub>,...<i>x<sub>n</sub>n</i><i>a<sub>n</sub></i> ta đợc


<i>a a</i>

1 2

...

<i>a</i>

<i>n</i> <i>n</i>

<i>a a a</i>

<sub>1 2</sub>

. ...

<i><sub>n</sub></i>


<i>n</i>



<sub></sub>

<sub> ( ®pcm )</sub>


</div>
<span class='text_page_counter'>(24)</span><div class='page_container' data-page=24>

<b>Mét sè bµi tËp: </b>


Bµi 1:


Chøng minh r»ng




<i>n</i>
<i>n</i>


1
2


1
....
2


1
1


1


2
2


2      <i>n</i><i>N</i>;<i>n</i>1 (1)
Bµi 2:


Cho <i>n N</i> vµ a+b > 0. Chøng minh r»ng


<i>n</i>


<i>b</i>
<i>a</i>







 


2  2


<i>n</i>
<i>n</i> <i><sub>b</sub></i>


<i>a </i> <sub> (1)</sub>


Bµi 3: Cho a,b là hai cạnh góc vuông của một tam giác vuông với c là cạnh huyền
Chứng minh rằng: <i>a</i>2<i>n</i> <i>b</i>2<i>n</i> <i>c</i>2<i>n</i> <i>n N</i>


<b>Dạng 7 - Phơng pháp áp dụng c¸c tÝnh chÊt cđa c¸c d·y tØ sè b»ng nhau</b>


<i>Đây là phơng pháp đặc trng cho học sinh THCS vì phơng pháp này áp dụng các </i>
<i>tính chất của dãy tỉ số bằng nhau đã đợc học ở lớp 7. Các tính chất đặc biệt thờng </i>
<i>gặp trong loại này ta cần lu ý nh:</i>


<b>KiÕn thøc:</b>




, ,


, , ,


<i>a</i> <i>a</i>


<i>a b c R</i>
<i>a b</i> <i>a b c</i>


<i>a</i> <i>c</i> <i>a</i> <i>a c</i> <i>c</i>


<i>a b c d R</i>



<i>b</i> <i>d</i> <i>b</i> <i>b d</i> <i>d</i>






  


  




     




<b>Mét sè vÝ dơ:</b>


VÝ dơ 1: Cho 3 sè d¬ng a,b,c chøng minh r»ng 1 a b c 2
a b b c c a






<b>Giải:</b>


Vì: a 1
a b  nªn:



a a a c


a b c a b a b c




 


    


T¬ng tù:


b b b a


a b c b c a b c




 


    


c c c b


a b c c a a b c




 



    


Cộng các bất đẳng thức trên lại ta đợc điều phải chứng minh.


</div>
<span class='text_page_counter'>(25)</span><div class='page_container' data-page=25>

Ví dụ 2: Cho a,b,c,d là các sè d¬ng, chøng minh r»ng:


A a b b c c d d a


a b c b c d c d a d a b


   


   


    


<b>Giải:</b>


Vì: a b 1
a b c






  nªn


a b a b a b d



a b c d a b c a b c d


   


 


       


T¬ng tù:




b c b c b c a


a b c d b c d a b c d


c d c d c d b


a b c d c d a a b c d


d a d a d a c


a b c d d a b a b c d


   


 


       



   


 


       


   


 


       


Cộng lại ta đợc 2<A<3. Suy ra A khụng th l s nguyờn .


<b>Bài tập áp dụng:</b>


Bài 1:


Cho a, b, c, d > 0 . Chøng minh r»ng:


1 <i>a</i> <i>b</i> <i>c</i> <i>a</i> 2


<i>a b c b c d</i> <i>c d a</i> <i>d a b</i>


    


       


Bµi 2:



Cho a;b;c;d là các số nguyên dơng thỏa mÃn : a + b = c + d =1000


tìm giá trị lớn nhất của


<i>d</i>
<i>b</i>
<i>c</i>
<i>a</i>


.


Bài 3:


Cho:


<i>b</i>
<i>a</i>


<


<i>d</i>
<i>c</i>


và b,d > 0 . Chøng minh r»ng


<i>b</i>
<i>a</i>


<



<i>d</i>
<i>c</i>
<i>d</i>
<i>b</i>


<i>cd</i>
<i>ab</i>






2
2


<b>D¹ng 8 </b>–<b> Phơng pháp dùng tam thức bậc hai</b>
<b>Kiến thức:</b>


Cho tam thức bËc hai <i>f</i> <i>x</i> <i>ax</i>2<i>bx</i><i>c</i>


NÕu 0 th× <i>a</i>.<i>f</i> <i>x</i> 0 <i>x R</i>


NÕu 0 th× <i>a</i>.<i>f</i> <i>x</i> 0


</div>
<span class='text_page_counter'>(26)</span><div class='page_container' data-page=26>

NÕu 0 th× <i>a</i>.<i>f</i> <i>x</i> 0 víi <i>x x</i><sub>1</sub> hc <i>x x</i><sub>2</sub> (<i>x </i><sub>2</sub> <i>x</i><sub>1</sub>)


<i>a</i>.<i>f</i> <i>x</i> 0 víi <i>x</i><sub>1</sub><i>x</i><i>x</i><sub>2</sub>


<b>Mét sè vÝ dô:</b>



VÝ dô 1: Chøng minh r»ng: <sub>x</sub>2<sub></sub><sub>5y</sub>2 <sub></sub> <sub>4xy 2x 6y 3 0</sub><sub></sub> <sub></sub> <sub></sub> <sub></sub> Víi mäi giá trị của
x,y.


<b>Giải:</b>


Đặt: f(x) x 2 (2y 1)2x 5y  2 6y 3


Ta cã:


2 2


2


(2y 1) 5y 6y 3


y 2y 2


( ) 1 2 1


0 f(x)>0


( ) 0


y R x,y R


D


D
d



D
d


     


   


   


  




 


   <sub></sub>  <sub></sub>


   


 


 


VÝ dô 2: Cho hai d·y sè: a1, a2 ,… an


B1, b2,… bn


Chøng minh r»ng:


n n n



2 2 2


i i i i


1 1 1


(

<sub></sub>

a b ) (

<sub></sub>

a )(

<sub></sub>

b ) (1)


<b>Gi¶i:</b>


Ta cã: (1)


n n n


2 2


i i i i


1 1 1


( a b ) ( a )( b ) 0 (2)


<sub></sub>

<sub></sub>

<sub></sub>



Đặt:


n n n


2 2 2



i i i i


1 1 1


f(x) (

<sub></sub>

a )X  2(

<sub></sub>

a b )X (

<sub></sub>

b )


Ta cã:


n


2


i i


1


f(X)

<sub></sub>

(a X b ) 0 Víi mäi X nªn tam thøc (X) cã<sub>D </sub><sub>' 0</sub>


Suy ra:


n n n


2 2 2


i i i i


1 1 1


(

<sub></sub>

a b )  (

<sub></sub>

a )(

<sub></sub>

b ) 0


Tức là (2) đúng nên (1) đúng.


Ví dụ 3:

<i>x y R</i>

,

<b>, chứng mih bất đẳng thức sau:</b>


</div>
<span class='text_page_counter'>(27)</span><div class='page_container' data-page=27>

<i>x y</i>2 42(<i>x</i>22)<i>y</i>24<i>xy x</i> 24<i>xy</i>3 (1)


<b>Gi¶i:</b>


(1)  (<i>y</i>21)2 2<i>x</i> 4 (1<i>y</i>  <i>y x</i>2) 4<i>y</i>20


Đặt <i>F x</i>( ) ( <i>y</i>21)2 2<i>x</i> 4 (1<i>y</i>  <i>y x</i>2) 4<i>y</i>2


 ' 4 (1<i>y</i>2  <i>y</i>2 2)  4 (<i>y y</i>2 21)2


<sub> </sub>

'

<i>16y</i>

2




' <sub>0</sub> <sub>( ) 0</sub>


,


<i>f x</i>
<i>x y R</i>
<i>y R</i>


 <sub></sub>





















<b>Bài tập áp dụng:</b>


Bài 1: Cho a,b,c,d tho· m·n b < c < d.


Chøng minh r»ng (a b c d)   2 8(ac bd) (1)


Bài 2: Cho các số a , b , c , d , p , q sao cho:


2 2 2 2 2 2


p q  a  b  c  d  0


Chøng minh r»ng:



2 2 2 2 2 2 2


(p  a  b )(q  c  d ) (pq ac bd) (1)


<b>Dạng 9 </b><b> Phơng pháp dùng tính chất bắc cầu</b>


<i>õy l phng phỏp chng minh bất đẳng thức sử dụng tính chất bắc cầu trong </i>
<i>Toỏn hc. </i>


<i>Chẳng hạn: a b</i> <i> và </i>

<i>b c</i>

<sub></sub>

<i> th× a c</i>
<b>Mét sè vÝ dơ:</b>


VÝ dơ 1:


Cho a, b, c ,d >0 tháa m·n a > c+d , b >c+d. Chøng minh r»ng: ab >ad+bc


<b>Gi¶i</b>


Ta cã:













<i>d</i>
<i>c</i>
<i>b</i>


<i>d</i>
<i>c</i>
<i>a</i>
















0
0
<i>c</i>
<i>d</i>
<i>b</i>


<i>d</i>


<i>c</i>
<i>a</i>


 (a-c)(b-d) > cd


</div>
<span class='text_page_counter'>(28)</span><div class='page_container' data-page=28>

 ab > ad + bc ®iỊu ph¶i chøng minh.
VÝ dơ 2:


Cho a,b,c>0 tháa m·n


3
5


2
2
2




<i>b</i> <i>c</i>


<i>a</i>


Chøng minh


<i>abc</i>
<i>c</i>
<i>b</i>
<i>a</i>



1
1
1
1






<b>Gi¶i</b>


Ta cã :( a + b – c)2<sub>= a</sub>2<sub>+ b</sub>2 <sub>+ c</sub>2 <sub>+ 2( ab – ac – bc) > 0</sub>


 ac + bc - ab



2
1


( a2 <sub>+ b</sub>2 <sub>+ c</sub>2<sub>) </sub>


 ac + bc – ab


6
5


1 Chia hai vÕ cho abc > 0 ta cã


<i>c</i>
<i>b</i>
<i>a</i>



1
1
1






<i>abc</i>


1


VÝ dô 3:


Cho 0<i>a b c</i>, , 1. Chøng minh r»ng:


<i>a</i>
<i>c</i>
<i>c</i>
<i>b</i>
<i>b</i>
<i>a</i>
<i>c</i>


<i>b</i>


<i>a</i>3 <sub>2</sub> 3 <sub>2</sub> 3 <sub>3</sub> 2 2 2


2      



<b>Gi¶i</b>


Do a < 1  2 1


<i>a</i> vµ
Ta cã

1 2

.1  0





 <i>a</i> <i>b</i>  1- b - <i>a</i>2 + <i><sub>a</sub></i>2<sub>b > 0</sub>


 1+<i><sub>a</sub></i>2 <i><sub>b</sub></i>2<sub> > </sub><i><sub>a</sub></i>2<sub> + b</sub>


mµ 0 < a,b < 1  a2 a , b3 2 b3


Từ đó ta suy ra 1+<sub>a b</sub>2 2 <sub>a</sub>3 <sub>b</sub>3


 


VËy <sub>a</sub>3 <sub>b</sub>3


 < <i>1 a b</i> 2 2


T¬ng tù ta cã:<sub>b</sub>3 <sub>c</sub>3


 <sub>1 </sub><i>b</i>2<i>c</i>





Vµ <sub>c</sub>3 <sub>a</sub>3


  1 <i>c</i>2<i>a</i>


Cộng các bất đẳng thức ta có: <sub>2</sub><i>a</i>3 <sub>2</sub><i>b</i>3 <sub>2</sub><i>c</i>3 <sub>3</sub> <i>a</i>2<i>b</i> <i>b</i>2<i>c</i> <i>c</i>2<i>a</i>









VÝ dô 4:


Cho 0<i>x y z</i>, , 1 Chøng minh r»ng:
a. 0 <i>x y z xy yz zx</i>    1


b. <i>x</i>2  <i>y</i>2 <i>z</i>2  1 <i>x y y z z x</i>2  2  2


<b>Gi¶i</b>


</div>
<span class='text_page_counter'>(29)</span><div class='page_container' data-page=29>

a. Ta cã: <i>x y z xy yz zx x</i>      (1 <i>y</i>) <i>y</i>(1 <i>z</i>)<i>z</i>(1 <i>x</i>) 0 (1)
Mặt khác: (1 <i>x</i>)(1 <i>y</i>)(1 <i>z</i>) 1  <i>x y xy yz zx xyz</i>     0


Suy ra: <i>x y z xy yz zx</i>      1 <i>xyz</i>1 (2)
Tõ (1) vµ (2) suy ra 0 <i>x y z xy yz zx</i>    1



b. Ta chøng minh: <i>x</i>2  <i>y</i>2 <i>z</i>2  <i>x y y z z x</i>2  2  2 1


Ta cã:


2 2 2 2 2 2 2<sub>(1</sub> <sub>)</sub> 2<sub>(1</sub> <sub>)</sub> 2<sub>(1</sub> <sub>)</sub>


(1 ) (1 ) (1 )


<i>x</i> <i>y</i> <i>z</i> <i>x y y z z x x</i> <i>y</i> <i>y</i> <i>z</i> <i>z</i> <i>x</i>


<i>x</i> <i>y</i> <i>y</i> <i>z</i> <i>z</i> <i>x</i>


          


     


V× ( <i>x</i>2 <i>x y</i>, 2 <i>y z</i>, 2 <i>z</i>)


1


<i>x y z xy yz zx</i>


       ( theo câu a).


<b>Bài tập áp dụng:</b>


Bài 1:


Cho 0<i>a b c d</i>, , , 1. Chøng minh r»ng:



(1 <i>a</i>)(1 <i>b</i>)(1 <i>c</i>)(1 <i>d</i>) 1  <i>a b c d</i>  


Bµi 2:


Cho 0<i>a b c</i>, , 2 tho¶ m·n <i>a b c</i>  3 Chøng minh r»ng:
<i><sub>a</sub></i>2 <i><sub>b</sub></i>2 <i><sub>c</sub></i>2 <sub>5</sub>


  


Bµi 3:


Cho a, b, c là độ dài 3 cạnh của tam giác có chu vi bằng 1. Chứng minh rằng:


2 2 2 <sub>4</sub> 1


2


<i>a</i> <i>b</i> <i>c</i>  <i>abc</i>


Bµi 4:


Cho a, b, c là độ dài 3 cạnh của tam giác có chu vi bằng 2. Chứng minh rằng:


2 2 2 <sub>2</sub> <sub>2</sub>


<i>a</i> <i>b</i> <i>c</i>  <i>abc</i>


<b>Dạng 10 </b>–<b> Phơng pháp dùng các bất đẳng thức trong tam giác</b>


<i>Đây là phơng pháp sử dụng các bất đẳng thức trong tam giác làm các giả thiết để </i>


<i>chứng minh các bất đẳng thức.</i>


</div>
<span class='text_page_counter'>(30)</span><div class='page_container' data-page=30>

1. Các bất đẳng thức trong tam giác:


Với a, b, c là 3 cạnh của một tam giác thì <i>a b c </i>, , 0




<i>b c</i> <i>a b c</i>
<i>a c b a c</i>
<i>a b c a b</i>


   


   


   


Nếu <i>a b c</i><sub>  thì số đo của 3 góc A, B, C cũng đúng với bất đẳng thức trên.</sub>
2. Công thức liên quan đến tam giác


2


2 2


2


<i>b c a</i>
<i>p a</i>



<i>a b c</i> <i>c a b</i>


<i>p</i> <i>p b</i>


<i>a b c</i>
<i>p c</i>


 


 





    


  <sub></sub>  




 


 





<b>Mét sè vÝ dơ:</b>



VÝ dơ 1:


Cho a, b, c lµ 3 cạnh của tam giác. Chứng minh rằng:


3 3 3 <sub>2</sub> 2<sub>(</sub> <sub>)</sub> 2<sub>(</sub> <sub>)</sub> 2<sub>(</sub> <sub>) (1)</sub>


<i>a</i> <i>b</i> <i>c</i>  <i>abc a b c</i>  <i>b c a</i> <i>c a b</i>


<b>Gi¶i</b>


Ta cã:


(1)


3 3 3 2 2 2


2 2 2 2 2


2 2 2


2 ( ) ( ) ( ) 0


( ) ( ) (2 ) ( ) 0


( )( ) ( ) ( ) 0


( )( )( ) 0 (2)


<i>a</i> <i>b</i> <i>c</i> <i>abc a b c</i> <i>b c a</i> <i>c a b</i>



<i>a a b</i> <i>b b a</i> <i>c ab a</i> <i>b</i> <i>c a b c</i>


<i>a b a b</i> <i>c a b</i> <i>c a b c</i>
<i>a b c a b c a b c</i>


          


          


        


       


Kết quả (2) ln đúng vì trong tam giác ta ln có.




0
0
0


<i>a b c</i> <i>a b c</i>


<i>a c b</i> <i>a c b</i>


<i>b c a</i> <i>b c a</i>


    



 


 


     


 


 <sub> </sub>  <sub> </sub> <sub></sub>


 


Vậy bất đẳng thức (1) đợc chứng minh.
Ví dụ 2:


</div>
<span class='text_page_counter'>(31)</span><div class='page_container' data-page=31>

Cho a;b;clà số đo ba cạnh của tam giác chøng minh r»ng:
a. a2 <sub>+ b</sub>2 <sub>+ c</sub>2 <sub>< 2(ab + bc + ac)</sub>


b. abc>(a + b - c).(b + c - a)( c + a - b)


<b>Giải</b>


Vì a, b, c là số đo 3 cạnh của tam giác nên ta cã





















<i>b</i>
<i>a</i>
<i>c</i>


<i>c</i>
<i>a</i>
<i>b</i>


<i>c</i>
<i>b</i>
<i>a</i>


0
0
0




















)
(


)
(


)
(


2
2
2


<i>b</i>
<i>a</i>


<i>c</i>
<i>c</i>


<i>c</i>
<i>a</i>
<i>b</i>
<i>b</i>


<i>c</i>
<i>b</i>
<i>a</i>
<i>a</i>


Cộng từng vế các bất đẳng thức trên ta có:
a2<sub>+b</sub>2<sub>+c</sub>2<sub>< 2(ab+bc+ac)</sub>


Ta cã a > b-c   <i><sub>a</sub></i>2 <i><sub>a</sub></i>2 <sub>(</sub><i><sub>b</sub></i> <i><sub>c</sub></i><sub>)</sub>2





 > 0


b > a-c   <i><sub>b</sub></i>2 <i><sub>b</sub></i>2 <sub>(</sub><i><sub>c</sub></i> <i><sub>a</sub></i><sub>)</sub>2





 > 0



c > a-b   2 2 ( )2 0



<i>c</i> <i>a</i> <i>b</i>
<i>c</i>


Nhân từng vế của đẳng thức lại ta đợc:


2

2

2


2 2 2 2 2 2


<i>a b c</i>

<i>a</i>

<i>b c</i>

 

<i>b</i>

<i>c a</i>

 

<i>c</i>

<i>a b</i>





 

 



 

 



 

 



2 2 2


2 2 2


. .


<i>a b c</i> <i>a b c</i> <i>b c a</i> <i>c a b</i>


<i>abc</i> <i>a b c b c a</i> <i>c a b</i>


       


       


VÝ dơ 3:


Trong Δ ABC có chu vi a + b +c = 2p ( a, b, c là độ dài 3 cạnh ).


CMR : <i><sub>p </sub></i>1<i><sub>a</sub></i> + <i><sub>p </sub></i>1 <i><sub>b</sub></i>+ <i><sub>p </sub></i>1 <i><sub>c</sub></i> ≥ 2 (


<i>a</i>


1


+


<i>b</i>


1


+


<i>c</i>


1


)



<b>Gi¶i</b>


Ta cã : p - a =


2


<i>a</i>
<i>c</i>
<i>b</i> 


> 0 ( vì b + c > a bất đẳng thức tam giác )


T¬ng tù : p - b > 0 ; p- c > 0.


Mặt khác : p - a + p - b = 2p - a - b = c
p - b + p - c = a


p - c + p - a = b


ta áp dụng tính chất của Bất đẳng thức: 1 1 4


</div>
<span class='text_page_counter'>(32)</span><div class='page_container' data-page=32>

<i>a</i>
<i>p </i>


1


+ <i><sub>p </sub></i>1<i><sub>b</sub></i> ≥ <sub>(</sub><i><sub>p</sub></i><sub></sub> <i><sub>a</sub></i><sub>)</sub><sub></sub>4<sub>(</sub><i><sub>p</sub></i><sub></sub> <i><sub>b</sub></i><sub>)</sub> =


<i>c</i>



4


<i>b</i>
<i>p </i>


1


+ <i><sub>p </sub></i>1 <i><sub>c</sub></i> ≥


<i>a</i>


4


<i>c</i>
<i>p </i>


1


+ <i><sub>p </sub></i>1<i><sub>a</sub></i> ≥


<i>b</i>


4


Cộng theo vế của bất đẳng thức ta có :


<i>a</i>
<i>p </i>


1



+ <i><sub>p </sub></i>1<i><sub>b</sub></i>+ <i><sub>p </sub></i>1 <i><sub>c</sub></i> ≥ 2 (


<i>a</i>


1


+


<i>b</i>


1


+


<i>c</i>


1


)


Dấu ‘=’ xảy ra khi Δ ABC đều
Ví dụ 4:


Cho a, b, c , là độ dài ba cạnh của một tam giác
(a+b-c)(b+c-a)(c+a-b) abc


<b>Gi¶i</b>


Bất đẳng thức về ba cạnh của tam giác :



2 2 2


0 ( )


<i>b c</i>  <i>a</i> <i>a</i>  <i>b c</i> <i>a</i>


2 2 2


0 ( )


<i>c a</i>  <i>b</i> <i>b</i>  <i>c a</i> <i>b</i>


2 2 2


0

(

)



<i>a b</i>

 

<i>c</i>

<i>c</i>

<i>a b</i>

<i>c</i>



Từ đó <i>a</i>2 (<i>b c</i> )2 <i>b</i>2  (<i>c a</i> )2 <i>c</i>2  (<i>a b</i> )2 <i>a b c</i>2 2 2


 (a + b – c)( a – b + c)( b – c +a)( b + c – a)( c – a + b)( c + a – b)
2 2 2


<i>a b c</i>


 (a + b – c)2<sub>( b + c – a)</sub>2<sub>( c + a – b)</sub>2 2 2 2


<i>a b c</i>






 (a + b – c)( b + c – a )( c + a – b) abc


V× a, b, c là 3 cạnh của một tam giác nên
0


0
0


<i>a b c</i>
<i>a c b</i>
<i>b c a</i>


  





  




   


vµ abc > 0


<b>Bài tập áp dụng: Cho a, b, c là độ dài 3 cạnh của tam giác</b>



Bµi 1:


</div>
<span class='text_page_counter'>(33)</span><div class='page_container' data-page=33>

Chøng minh r»ng:


NÕu víi <i>a b c</i>  th× (<i>a b c</i>  )2 9<i>bc</i>


Bµi 2:


Chøng minh r»ng:


(<i>a b c b c a a c b</i>  )(   )(   )<i>abc</i>


Bµi 3:


Chøng minh r»ng: Víi mäi p, q sao cho p + q = 1 th× <i>pa</i>2 <i>qb</i>2  <i>pqc</i>2


Bµi 4:


Cho a, b, c là độ dài 3 cạnh của một tam giác với a < b < c. Chứng minh rằng:


3<sub>(</sub> 2 2<sub>)</sub> 3<sub>(</sub> 2 2<sub>)</sub> 3<sub>(</sub> 2 2<sub>) 0</sub>


<i>a b</i>  <i>c</i> <i>b c</i>  <i>a</i> <i>c a</i>  <i>b</i> 


Bµi 5:


Chứng minh rằng: với a, b,c là độ dài 3 cạnh của một tam giác thì ta có:


2 2 2 3 3 3



( ) ( ) ( ) (1)


<i>a b c</i> <i>b c a</i> <i>c a b</i> <i>a</i> <i>b</i> <i>c</i>


<b>Dạng 11 </b>–<b> Phơng pháp đổi biến số</b>


<i>Khi ta gặp một số bất đẳng thức có biến phức tạp thì ta có thể dùng phơng pháp </i>
<i>đổi biến số để đa các bất đẳng thức cần chứng minh về dạng đơn giản hơn, tức là </i>
<i>ta đặt các biến mới biểu thị đợc các bién cũ sao cho biến mới có thể gọn hơn hoặc </i>
<i>dễ chứng minh hơn. Sau khi đổi biến số ta sử dụng các phơng pháp chứng minh ở </i>
<i>trên để chứng minh bất đẳng thức.</i>


<i>Phơng pháp lợng giác cũng là một dạng của phơng pháp đổi biến số.</i>
<b>Một số ví dụ:</b>


VÝ dô 1:


Cho a,b,c > 0 Chøng minh r»ng:


2
3






 <i>a</i> <i>b</i>



<i>c</i>
<i>a</i>
<i>c</i>


<i>b</i>
<i>c</i>
<i>b</i>


<i>a</i>


(1)


<b>Giải</b>


Đặt x=b+c ; y=c+a ;z= a+b ta cã a=


2


<i>x</i>
<i>z</i>
<i>y</i> 


; b =


2


<i>y</i>
<i>x</i>
<i>z</i> 



; c =


2


<i>z</i>
<i>y</i>
<i>x</i> 


Ta cã (1)  <i>y</i><sub>2</sub><i>z<sub>x</sub></i> <i>x</i> <i>z</i><sub>2</sub><i>x<sub>y</sub></i> <i>y</i> <i>x</i><sub>2</sub><i>y<sub>z</sub></i> <i>z</i>


2
3


  1  1  13
<i>z</i>


<i>y</i>
<i>z</i>
<i>x</i>
<i>y</i>
<i>z</i>
<i>y</i>
<i>x</i>
<i>x</i>


</div>
<span class='text_page_counter'>(34)</span><div class='page_container' data-page=34>

 (  )(  )(  )6
<i>z</i>
<i>y</i>
<i>y</i>


<i>z</i>
<i>z</i>
<i>x</i>
<i>x</i>
<i>z</i>
<i>y</i>
<i>x</i>
<i>x</i>
<i>y</i>


Bất đẳng thức cuối cùng đúng vì (  2;
<i>y</i>
<i>x</i>
<i>x</i>
<i>y</i>


2


<i>z</i>
<i>x</i>
<i>x</i>
<i>z</i>


; 2
<i>z</i>
<i>y</i>
<i>y</i>
<i>z</i>


) điều phải



chøng minh.
VÝ dơ 2:


Cho a,b,c > 0 vµ a+b+c < 1. Chøng minh r»ng:


9
2
1
2


1
2


1


2
2


2 







 <i>bc</i> <i>b</i> <i>ac</i> <i>c</i> <i>ab</i>


<i>a</i> (1)



<b>Giải</b>


Đặt x = <i>a</i>2 2<i>bc</i>


; y = <i>b</i>2 2<i>ac</i> ; z = <i>c</i>2 2<i>ab</i>


Ta cã <i>x</i><i>y</i><i>z</i><i>a</i><i>b</i><i>c</i>21


(1)  1119
<i>z</i>
<i>y</i>


<i>x</i> Víi x+y+z < 1 vµ x ,y,z > 0


Theo bất đẳng thức Cơsi ta có:
<i>x</i><i>y</i><i>z</i><sub>3.</sub>3 <i><sub>xyz</sub></i>


  
<i>z</i>
<i>y</i>
<i>x</i>


1
1
1


3. .3 1


<i>xyz</i>



  . 1 1 1<sub></sub>9














<i>z</i>
<i>y</i>
<i>x</i>
<i>z</i>
<i>y</i>
<i>x</i>


Mµ x+y+z < 1


VËy 111 9
<i>z</i>
<i>y</i>


<i>x</i> điều phải chứng minh.


Ví dụ 3:



<b>Cho a, b, c là các số thực dơng. Chứng minh rằng: </b>

3



2



<i>a</i>

<i>b</i>

<i>c</i>



<i>b c c a a b</i>


<b>Gi¶i</b>


Đây là ví dụ 1 nhng ta sử dụng cách đổi biến khác:


Ta đặt


2


2


2


<i>y z x</i>
<i>a</i>


<i>x b c</i>


<i>x z y</i>


<i>y c a</i> <i>b</i>


<i>z a b</i> <i><sub>x y z</sub></i>



<i>c</i>


 




 


 <sub></sub>


 


 


   


 


 <sub> </sub> 


 <sub></sub> <sub> </sub>






</div>
<span class='text_page_counter'>(35)</span><div class='page_container' data-page=35>

Bất đẳng thức 1 3



2 2


<i>y z x</i> <i>x z y</i> <i>x y z</i>


<i>x</i> <i>y</i> <i>z</i>


       


 <sub></sub>   <sub></sub> 


 


2 . 2 . 2 . 6


<i>x</i> <i>y</i> <i>y</i> <i>z</i> <i>z</i> <i>x</i> <i>x y</i> <i>y z</i> <i>z x</i>


<i>y</i> <i>x</i> <i>z</i> <i>y</i> <i>x</i> <i>z</i> <i>y x</i> <i>z y</i> <i>x z</i>


     


 <sub></sub>  <sub></sub><sub></sub>  <sub></sub><sub></sub>  <sub></sub>    


 


   


(đúng).


Vậy Bất đẳng thức đợc chứng minh.
Dấu “=” xảy ra  <i>a b c</i> 



VÝ dơ 4:


Cho c¸c sè thùc d¬ng a, b, c sao cho abc = 1
Chøng minh r»ng:


1 1 1


1 1 1 1


<i>a</i> <i>b</i> <i>c</i>


<i>b</i> <i>c</i> <i>a</i>


     


      


     


     


<b>Gi¶i</b>


Do <i>abc </i>1 nên ta có thể đặt:


<i>x</i>
<i>a</i>


<i>y</i>


<i>y</i>
<i>b</i>


<i>z</i>
<i>z</i>
<i>c</i>


<i>x</i>

















víi <i>x y z </i>, , 0.


Nên bất đẳng thức ta có thể viết lại nh sau: <i>x</i> 1 <i>z</i> <i>y</i> 1 <i>x</i> <i>z</i> 1 <i>y</i> 1


<i>y</i> <i>y</i> <i>z</i> <i>z</i> <i>x</i> <i>x</i>



     


      


     


   


 


<b> </b> <i>xyz</i> (<i>x y z y z x z x y</i>  )(   )(   )<b> (Ta đã chứng minh đợc)</b>


Vậy BĐT đã đợc chứng minh. Dấu “=” xảy ra  <i>a b c</i> 1


<b>Bài tập áp dụng:</b>


Bài 1:


Chng minh rng ; vi mọi số thực x, y ta có bất đẳng thức:




4
1
)
1
(
)
1
(



)
1
)(
(


4
1


2
2
2
2


2
2
2
2









<i>y</i>
<i>x</i>


<i>y</i>


<i>x</i>
<i>y</i>
<i>x</i>


Bài 2:


Cho x, y, z là các số dơng tho¶ m·n: 


<i>x</i>


1


<i>y</i>
1


+


<i>z</i>


1


</div>
<span class='text_page_counter'>(36)</span><div class='page_container' data-page=36>

CMR: <i><sub>x</sub></i> <i><sub>y</sub></i> <i><sub>z</sub></i>




2


1


+ <i><sub>x</sub></i> <i><sub>y</sub></i> <i><sub>z</sub></i>




2


1


+ <i><sub>x</sub></i> <i><sub>y</sub></i>1 <sub>2</sub><i><sub>z</sub></i>




 1


(Đại học khối A năm 2005)
Bài 3:


Cho a, b, c l c¸c sè thùc dà ¬ng tho¶ m·n abc=1 .


<b> CMR : </b> <sub>3</sub> 1 <sub>3</sub> 1 <sub>3</sub> 1 3


( ) ( ) ( ) 2


<i>a b c</i> <i>b c a</i> <i>c a b</i> 


Bµi 4:


Cho x, y, z >0 tho¶ m·n <i>x y z</i>  1<b>. CMR </b>

1

4 9

36



<i>x</i>

<i>y</i>

<i>z</i>



Bµi 5:



<b>Cho x, y, z là các số thực dương thoả mãn: </b>

<i>xyz</i>

   

<i>x y z</i>

2

<b>. </b>


<b> CMR: </b>

3



2



<i>x</i>

<i>y</i>

<i>z</i>

<i>xyz</i>



<b>Dạng 12 - Phơng pháp làm trội (chứng minh bất đẳng thức có n số hạng)</b>


<i>Dùng các tính bất đẳng thức để đa một vế của bất đẳng thức về dạng tính đợc tổng</i>
<i>hữu hạn hoặc tích hữu hạn.</i>


<i>Phơng pháp chung để tính tổng hữu hạn : S = u</i>1<i>u</i>2....<i>un</i>


<i>Ta cố gắng biến đổi số hạng tổng quát uk</i> <i> về hiệu của hai số hạng liên tiếp nhau</i>


1



 <i>k</i> <i>k</i>
<i>k</i> <i>a</i> <i>a</i>


<i>u</i>


<i>Khi đó : S = </i>

<i>a</i>1 <i>a</i>2

<i>a</i>2 <i>a</i>3

....

<i>an</i> <i>an</i>1

<i>a</i>1 <i>an</i>1


<i>Phơng pháp chung về tính tích hữu hạn:</i>


<i>P = u</i>1<i>u</i>2....<i>un</i>


<i>Bin i cỏc s hng uk về thơng của hai số hạng liên tiếp nhau:</i>


<i>k</i>


<i>u</i> <i>=</i>


1


<i>k</i>
<i>k</i>
<i>a</i>


<i>a</i>


<i>Khi đó P = </i>


1
1
1
3


2
2


1<sub>.</sub> <sub>...</sub>








<i>n</i>
<i>n</i>


<i>n</i>
<i>a</i>


<i>a</i>
<i>a</i>


<i>a</i>
<i>a</i>


<i>a</i>
<i>a</i>
<i>a</i>


<b>Mét sè vÝ dơ:</b>


</div>
<span class='text_page_counter'>(37)</span><div class='page_container' data-page=37>

VÝ dô 1:


Chøng minh r»ng: .... 1 2

1 1


3


1
2
1



1     <i>n</i> 


<i>n</i> Với n là số nguyên


<b>Giải:</b>


Ta có:


<i>k</i> <i>k</i>



<i>k</i>
<i>k</i>
<i>k</i>


<i>k</i>  12 1
2


2
2
1


Khi cho k chạy từ 1 đến n ta có
1 > 2

2  1



3 2



2
2
1






………


<i>n</i> <i>n</i>



<i>n</i> 2 1


1


Cộng từng vế các bất đẳng thức trên ta có:


1 1



2
1
....
3
1
2
1


1     <i>n</i> 


<i>n</i>


VÝ dô 2:



Chøng minh r»ng: 1 2


1 2








<i>n</i>
<i>k</i> <i>k</i>


<i>n Z</i>


<b>Gi¶i:</b>


Ta cã


<i>k</i>  <i>k</i> <i>k</i>


<i>k</i>
<i>k</i>


1
1
1
1
1
1



2  <sub></sub>  <sub></sub> 


Cho k chạy từ 2 đến n ta có


1
1
....
3


1
2


1


1
1
1
1


...
...


3
1
2
1
3


1



2
1
1
2


1


2
2


2
2
2
2



















<i>n</i>
<i>n</i>
<i>n</i>
<i>n</i>


VËy 1 2


1 2








<i>n</i>
<i>k</i> <i>k</i>


</div>
<span class='text_page_counter'>(38)</span><div class='page_container' data-page=38>

Chøng minh r»ng:


1 1 1


1 .... 2 1 ,


2 3 <i>n</i> <i>n</i> <i>n Z</i>





       


<b>Gi¶i:</b>


Ta cã:


1 2 2


2( 1)


2 1 <i>k</i> <i>k</i> <i>k Z</i>


<i>k</i> <i>k</i> <i>k</i> <i>k</i>




      


 


Do đó:
1 1


1


2( 2 1)


2
1



2( 3 2)


3
...
1


2( <i>n</i> <i>n</i> 1)


<i>n</i>




 


 


  


VËy:


1 1 1


1 ... 1 2 2 2 1


2 3 <i>n</i> <i>n</i> <i>n</i>


        


VÝ dô 4:



Chøng minh r»ng:


1 3 5 2 1 2


. . ... ,


2 4 6 2 2 1


<i>n</i>


<i>n Z</i>


<i>n</i> <i>n</i>






  




<b>Gi¶i:</b>


Ta cã:


2 2 2 2


2



2 2 2 2


2 2 2


2 2


1 3 5 (2 1)


. . ...


2 4 6 (2 )


1 3 (2 1) 1


. ....


2 1 4 1 (2 1)(2 1) 2 1


<i>n</i>
<i>A</i>


<i>n</i>
<i>n</i>


<i>n</i> <i>n</i> <i>n</i>








 


    


VËy: 1 3 5 2. . ... 1 2


2 4 6 2 2 1


<i>n</i>


<i>n</i> <i>n</i>





 .


</div>
<span class='text_page_counter'>(39)</span><div class='page_container' data-page=39>

<b>Bài tập áp dụng:</b>


Bài 1:


Chứng minh BĐT sau:


a. 1 1 ... 1 1


1.3 3.5  (2<i>n</i> 1).(2<i>n</i>1)  2


b. 1 1 1 ... 1 2



1.2 1.2.3 <i>1.2.3...n</i>


    


<b>\</b>


<b>Phần III ứng dụng của Bất đẳng thức.</b>–


<i>Bất đẳng thức đợc ứng dụng rộng rãi nhiều trong việc tìm GTLN, GTNN, giải </i>
<i>ph-ơng trình và hệ phph-ơng trình, dùng để giải phph-ơng trình nghiệm nguyên và rất nhiều </i>
<i>ứng dụng khác nữa.</i>


<b>I - Dùng BĐT để tìm GTLN và GTNN</b>


<i>KiÕn thøc : NÕu f(x) </i><i> m thì f(x) có giá trị nhỏ nhất là m.</i>
<i> NÕu f(x) </i><i> M thì f(x) có giá trị lớn nhất lµ M.</i>


<i>Ta thờng hay áp dụng các bất đẳng thức thông dụng nh: Côsi, Bunhiacôpxki , bất </i>
<i>đẳng thức chứa dấu giá trị tuyệt đối, kiểm tra trờng hợp xảy ra dấu đẳng thức để </i>
<i>tìm cực trị. Tìm cực trị của một biểu thức có dạng là đa thức , ta hay sử dụng phơng</i>
<i>pháp biến đổi tơng đơng , đổi biến số , một số bất đẳng thức… Tìm cực trị của một </i>
<i>biểu thức có chứa dấu giá trị tuyệt đối , ta vận dụng các bất đẳng thức chứa dấu </i>
<i>giá trị tuyệt đối</i>


<i>Chó ý: </i> <i>A</i>  <i>B</i> <i>A</i><i>B</i>


<i>X¶y ra dÊu '' = '' khi AB </i><i> 0</i>


0



<i>A</i> <i><sub> DÊu ''= '' x¶y ra khi A = 0</sub></i>
<b>Mét sè vÝ dô:</b>


VÝ dô 1:


</div>
<span class='text_page_counter'>(40)</span><div class='page_container' data-page=40>

b. Tìm giá trị nhỏ nhất của biểu thøc :
B = - x2<sub> - y</sub>2<sub> + xy + 2x +2y</sub>


<b>Gi¶i</b>


Ta cã:


a.A = (x2<sub> + x)(x</sub>2<sub> + x - 4) . Đặt : t = x</sub>2<sub> + x – 2</sub>


=> A = (t - 2)(t + 2) = t2 <sub>- 4 </sub><sub></sub><sub> - 4</sub>


DÊu b»ng x¶y ra khi : t = 0  x2<sub> + x - 2 = 0</sub>


(x - 2)(x + 2) = 0  x = -2 ; x = 1
=> min A = - 4 khi x = -2 ; x = 1
b. Tơng tự


Ví dụ 2: Tìm giá trị nhá nhÊt cđa biĨu thøc
a. C = 2<i>x</i> 3  2<i>x</i> 1


b. D = 2 3 2 6








<i>x</i> <i>x</i> <i>x</i>


<i>x</i> <sub> </sub>


c. E = <i>x</i> 1 <i>x</i> 2  <i>x</i> 3 <i>x</i> 4


<b>Giải</b>


áp dụng BĐT : <i>A</i> <i>B</i> <i>A</i><i>B</i>


DÊu '' = ''x¶y ra khi AB  0


=> C = 2<i>x</i> 3 1 2<i>x</i> 2<i>x</i> 31 2<i>x</i>  2 2


DÊu '' = '' x¶y ra khi (2x - 3)(1 - 2x)  0 


2
3
2


1

<i>x</i>


VËy minC = 2 khi


2


3
2


1

<i>x</i>


b, T¬ng tù : minD = 9 khi : -3  x  2
c. T¬ng tù: minE = 4 khi : 2  x  3


VÝ dơ 3:


Cho ba sè d¬ng x , y , z tho¶ m·n :


<i>x</i>



1


1


+ <i><sub>y</sub></i>



1


1


+



<i>z</i>



1


1


2


Tìm giá trị lớn nhất của tích : P = xyz.


<b>Gi¶i</b>


<i>x</i>



1


1


 (1 - <i><sub>y</sub></i>

1


1


) + ( 1 -


<i>z</i>




1


1


) =


<i>y</i>
<i>y</i>


1 + <i>z</i>


<i>z</i>




1  2 (1 <i>y</i>)(1 <i>z</i>)
<i>yz</i>





T¬ng tù : <i><sub>y</sub></i>



1


1



 2


)
1
)(
1


( <i>x</i> <i>z</i>


<i>zx</i>



</div>
<span class='text_page_counter'>(41)</span><div class='page_container' data-page=41>



<i>z</i>



1


1


 2 <sub>(</sub><sub>1</sub> <i><sub>x</sub>xy</i><sub>)(</sub><sub>1</sub> <i><sub>y</sub></i><sub>)</sub>



Từ đó suy ra : P = xyz 


8
1



MaxP =


8
1


khi x = y = z =


2
1


VÝ dô 4:


Cho G = <i>yz</i> <i>x</i>1<i>zx</i> <i><sub>xyz</sub>y</i> 2<i>xy</i> <i>z</i> 3


Tìm giá trị lớn nhất của G.


<b>Gi¶i</b>


Tập xác định : x  1 ; y  2 ; z  3


Ta cã : G = <i>x 1<sub>x</sub></i> + <i>y<sub>y</sub></i> 2 + <i>z<sub>z</sub></i> 3


Theo BĐT Côsi ta có : <i>x</i> 1<i>x</i><sub>2</sub>11 =>


<i>x</i>
<i>x 1</i>


2
1




T¬ng tù :


2
2


1
2





<i>y</i>
<i>y</i>


;


3
2


1
3




<i>z</i>
<i>z</i>


=> G 



3
2


1
2
2


1
2
1





VËy MaxG =


3
2


1
2
2


1
2
1





 đạt đợc khi x = 2 ; y = 2 ; z = 6


<b>Bµi tập áp dụng:</b>


Bài 1:


Tìm giá trị lớn nhất của:


S = xyz.(x+y).(y+z).(z+x) víi x,y,z > 0 vµ x+y+z =1


Bµi 2: Cho xy+yz+zx = 1. Tìm giá trị nhỏ nhất của <i><sub>x</sub></i>4 <i><sub>y</sub></i>4 <i><sub>z</sub></i>4




Bài 3:Tìm giá trÞ nhá nhÊt cđa biĨu thøc A = x2<sub> – 2xy + 3y</sub>2<sub> – 2x – 10y + 20</sub>


Bµi 4:Tìm giá trị lớn nhất , nhỏ nhất của phân thøc D =


1
1


2
2





<i>x</i>


<i>x</i>


<i>x</i>


</div>
<span class='text_page_counter'>(42)</span><div class='page_container' data-page=42>

<b>II </b>–<b> Dùng BĐT để giải phơng trình và hệ phơng trình</b>


<i>Nhờ vào các tính chất của bất đẳng thức , các phơng pháp chứng minh bất đẳng </i>
<i>thức , ta biến đổi hai vế ( VT , VP ) của phơng trình sau đó suy luận để chỉ ra </i>
<i>nghiệm của phơng trình. Nếu VT = VP tại một hoặc một số giá trị nào đó của ẩn </i>
<i>( thoả mãn TXĐ) => phơng trình có nghiệm.</i>


<i>NÕu VT > VP hc VT < VP tại mọi giá trị của ẩn</i>


<i>phơng trình vô nghiƯm</i>


<i>Cịn đối với hệ phơng trình ta dùng bất đẳng thức để biến đổi từng phơng trình của </i>
<i>hệ , suy luận và kết luận nghiệm. Biến đổi một phơng trình của hệ , sau đó so sánh </i>
<i>với phơng trình cịn lại , lu ý dùng các bất đẳng thc quen thuc.</i>


Một số ví dụ:
Ví dụ 1:


Giải phơng trình : 13 <i>x</i> 1 + 9 <i>x</i>1 = 16x


<b>Giải</b>


Điều kiện : x  1 (*)


áp dụng bất đẳng thức Côsi ta có :
13 <i>x</i> 1 + 9 <i>x</i>1


= 13.2. 1



2
1




<i>x</i> + 3.2. 1


2
3




<i>x</i> <sub></sub> 13( x - 1 +


4
1


) + 3(x + 1 +


4
9


) = 16x


DÊu '' = '' x¶y ra

















2
3
1


2
1
1


<i>x</i>
<i>x</i>


 x =


4
5


thoả mÃn (*)


Phơng trình (1) có nghiƯm  dÊu '' = '' ë (2) x¶y ra



VËy (1) cã nghiƯm x =


4
5


.


VÝ dơ 2:


Gi¶i phơng trình : 2 <i>x</i> 3 + 5  2<i>x</i> - x2 + 4x - 6 = 0 (*)


<b>Giải</b>


TXĐ :


2
5
2


3

<i>x</i>


(*) 2 <i>x</i> 3 + 5  2<i>x</i> = x2<sub> - 4x + 6</sub>


VP = (x - 2)2<sub> + 2 </sub><sub></sub><sub> 2 , dÊu '' = '' x¶y ra khi x = 2</sub>


</div>
<span class='text_page_counter'>(43)</span><div class='page_container' data-page=43>

=> víi x = 2 ( thoả mÃn TXĐ ) thì VT = VP = 2
=> phơng trình (*) có nghiệm x = 2



Ví dụ 3:


Giải phơng trình : 6 <i>x</i> + <i>x</i>2 = x2 - 6x + 13


<b>Giải</b>


TXĐ : -2  x  6
VP = (x - 3)2<sub> + 4 </sub>


 4 . DÊu '' = '' x¶y ra khi x = 3


VT2<sub> = (</sub> <sub>6</sub><sub></sub> <i><sub>x</sub></i><sub>.1 + </sub> <i><sub>x</sub></i><sub></sub><sub>2</sub><sub>.1)</sub>2 <sub></sub><sub> (6 - x + x + 2)(1 + 1) = 16</sub>


=> VT  4 , dÊu '' = '' x¶y ra khi 6 <i>x</i> = <i>x</i>2  x = 2


=> không có giá trị nào của x để VT = VP => Phơng trình vơ nghiệm
Ví dụ 4:


Gi¶i hƯ phơng trình:


3 2


2 2 2


2

4

3 0 (1)



2

0

(2)



<i>x</i>

<i>y</i>

<i>y</i>




<i>x</i>

<i>x y</i>

<i>y</i>



 









<b>Gi¶i</b>


(1)  x3<sub> = - 1 - 2(y - 1)</sub>2<sub>  x</sub>3 <sub></sub><sub> - 1  x </sub><sub></sub><sub> - 1 . (*)</sub>


(2)  x2 <sub></sub>
2


1
2


<i>y</i>
<i>y</i>


  1 ( v× 1 + y


2 <sub></sub><sub> 2y)  -1 </sub><sub></sub><sub> x </sub><sub></sub><sub> 1 (**)</sub>


Tõ (*) vµ (**) => x = -1 . Thay x = -1 vµo (2) ta cã : y = 1
=> HƯ phơng trình có nghiệm duy nhất : x = -1 ; y = 1


Ví dụ 5:Giải hệ phơng trình :
















<i>xyz</i>
<i>z</i>
<i>y</i>
<i>x</i>


<i>z</i>
<i>y</i>
<i>x</i>


4
4
4


1



<b>Giải</b>


áp dụng : BĐT : <sub>a</sub>2 <sub>b</sub>2 <sub>2ab</sub>


  dÊu '' = '' x¶y ra khi a = b


Ta cã : x4 y42x y ; y2 2 4 z4 2y z ; z2 2 4 x4 2z x2 2


4 4 4 2 2 2 2 2 2


x y z x y y z z x (*)




Mặt khác: x y2 2 y z2 2 2x yz2




2 2 2 2 2


2 2 2 2 2


y z z x 2xy z


x y z x 2xyz


 


</div>
<span class='text_page_counter'>(44)</span><div class='page_container' data-page=44>

2 2 2 2 2 2



2 2 2 2 2 2


2(x y y z z x ) 2xyz(x y z) 2xyz


x y y z z x xyz (**)


      


   


Tõ (*) vµ (**) => x4<sub> + y</sub>4 <sub></sub> <sub>x</sub>4 <sub></sub><sub>y</sub>4 <sub></sub><sub>z</sub>4 <sub>xyz</sub>


DÊu '' = '' x¶y ra khi : x = y = z mµ x + y + z = 1 nªn : x = y = z =


3
1


Vậy hệ phơng trình có nghiệm : x = y = z =


3
1


<b>Bài tập áp dụng:</b>


Bài 1:Giải hệ phơng trình:




2 3 <sub>14</sub> <sub>(1)</sub>



1 1 1


( )( ) 1 (2)


2 3 6 2 3 6


<i>x y</i> <i>z</i>


<i>x</i> <i>y</i> <i>z</i>


<i>x</i> <i>y</i> <i>z</i>


   





    





(với x, y, z > 0)


Bài 2: Giải phơng tr×nh sau:


<sub>4 3</sub><i><sub>x</sub></i>2 <sub>6</sub><i><sub>x</sub></i> <sub>19</sub> <sub>5</sub><i><sub>x</sub></i>2 <sub>10</sub><i><sub>x</sub></i> <sub>14 4 2</sub><i><sub>x x</sub></i>2


   



Bài 3: Giải phơng trình:

<i><sub>x</sub></i>

<sub></sub>

<sub>2</sub>

<sub></sub>

<i><sub>x</sub></i>

2

<sub></sub>

<sub>4</sub>

<i><sub>y</sub></i>

2

<sub></sub>

<sub>4</sub>

<i><sub>y</sub></i>

<sub></sub>

<sub>3</sub>


Bài 4:Giải hệ phơng trình sau:


<sub>4</sub> <i>x y z</i><sub>4</sub> <sub>4</sub> 1


<i>x</i> <i>y</i> <i>z</i> <i>xyz</i>


  




  




Bµi 5: Giải hệ phơng trình sau:


2


2


4 8


2


<i>xy</i> <i>y</i>


<i>xy</i> <i>x</i>



   




 


(1)


(2)


<b>III </b>–<b> Dùng BĐT để giải phơng trình nghiệm nguyên</b>
<b>Một số vớ d:</b>


Ví dụ 1: Tìm các số nguyên x,y,z thoả m·n


<i>x</i>

2

<sub></sub>

<i>y</i>

2

<sub></sub>

<i>z</i>

2

<sub></sub>

<i>xy</i>

<sub></sub>

3

<i>y</i>

<sub></sub>

2

<i>z</i>

<sub></sub>

3



<b>Giải</b>


Vì x,y,z là các số nguyên nên


<i>x</i>

2

<sub></sub>

<i>y</i>

2

<sub></sub>

<i>z</i>

2

<sub></sub>

<i>xy</i>

<sub></sub>

3

<i>y</i>

<sub></sub>

2

<i>z</i>

<sub></sub>

3



</div>
<span class='text_page_counter'>(45)</span><div class='page_container' data-page=45>



2 2 2


2 2



2 2


3 2 3 0


3


3 3 2 1 0


4 4


<i>x</i> <i>y</i> <i>z</i> <i>xy</i> <i>y</i> <i>z</i>


<i>y</i> <i>y</i>


<i>x</i> <i>xy</i> <i>y</i> <i>z</i> <i>z</i>


       


   


 <sub></sub>   <sub></sub><sub></sub>   <sub></sub>   


   




2 2


2



3 1 1 0


2 2


<i>y</i> <i>y</i>


<i>x</i> <i>z</i>


   


 <sub></sub>  <sub></sub>  <sub></sub>  <sub></sub>   


   


(*)


<sub></sub>

<sub></sub>



2 2


2


3 1 1 0


2 2


<i>y</i> <i>y</i>


<i>x</i> <i>z</i>



   


     


   


   


<i>x y R</i>, 




2 2


2


3 1 1 0


2 2


<i>y</i> <i>y</i>


<i>x</i> <i>z</i>


   


 <sub></sub>  <sub></sub>  <sub></sub>  <sub></sub>   


   



0



2

<sub>1</sub>



1 0

2



2



1


1 0



<i>y</i>


<i>x</i>



<i>x</i>


<i>y</i>



<i>y</i>


<i>z</i>


<i>z</i>
















<sub></sub>

 

<sub></sub>



<sub> </sub>











Các số x,y,z phải tìm là


1
2
1
<i>x</i>
<i>y</i>
<i>z</i>












Ví dụ 2:


Tìm các cặp số nguyên thoả mÃn phơng trình:


<i>x</i>

<i>x</i>

<i>y</i>

<b> (*)</b>


<b>Gi¶i</b>


(*) Víi x < 0 , y < 0 thì phơng trình không có nghÜa
(*) Víi x > 0 , y > 0


Ta cã

<i><sub>x</sub></i>

<sub></sub>

<i><sub>x</sub></i>

<sub></sub>

<i><sub>y</sub></i>

<b> </b><sub></sub> <i><sub>x</sub></i><sub></sub> <i><sub>x</sub></i> <sub></sub><i><sub>y</sub></i>2


2

<sub>0</sub>



<i>x</i>

<i>y</i>

<i>x</i>



</div>
<span class='text_page_counter'>(46)</span><div class='page_container' data-page=46>

Đặt <i>x</i> <i>k</i> (k nguyên dơng vì x nguyên dơng )


Ta cã <i>k k</i>.( <sub></sub>1)<sub></sub><i>y</i>2


Nhng

<i>k</i>

2

<sub></sub>

<i>k k</i>

<sub></sub>

<sub></sub>

1

<sub> </sub>

<sub></sub>

<i>k</i>

<sub></sub>

1

<sub></sub>

2
 <i>k</i> <i>y k</i> 1


Mà giữa k và k+1 là hai số nguyên dơng liên tiếp không tồn tại một số nguyên
d-ơng nào cả nên không có cặp số nguyên dd-ơng nào thoả mÃn phd-ơng trình



Vậy phơng trình có nghiệm duy nhất là :

0



0



<i>x</i>


<i>y</i>









<b>Bài tập áp dụng:</b>


Bài 1: Tìm nghiệm nguyên dơng của phơng trình: 1 1 1 2


<i>x</i> <i>y</i><i>z</i> 


<b>Phần IV - giải và hớng dẫn giải BT áp dụng</b>
<b>Dạng 1 </b>–<b> Dựa vào định nghĩa v cỏc phộp bin i tng ng</b>


Bài 1:


<b>Giải</b>


Ta có: (<i>a b</i> )2  0 2<i>ab a</i> 2 <i>b</i>2





2 2 2 2


2 2 2


2 2 2


2 2


2 2 2 2


2 2( )


( ) 2( )


2 2( ) ( 2)


2


2 ( )


<i>a</i> <i>b</i> <i>ab</i> <i>a</i> <i>b</i>


<i>a b</i> <i>a</i> <i>b</i>


<i>a</i> <i>b</i> <i>Do a b</i>


<i>a</i> <i>b</i>


<i>a</i> <i>b</i>



    


   


    


  


  


T¬ng tù: (<i>a</i>2  <i>b</i>2 2)  0 2<i>a b</i>2 2 <i>a</i>4 <i>b</i>4


4 4 2 2 4 4


2 2 2 4 4


2 2( )


( ) 2( )


<i>a</i> <i>b</i> <i>a b</i> <i>a</i> <i>b</i>


<i>a</i> <i>b</i> <i>a</i> <i>b</i>


    


   


VËy: 22 (<i>a</i>2 <i>b</i>2 2) 2(<i>a</i>4 <i>b</i>4)



</div>
<span class='text_page_counter'>(47)</span><div class='page_container' data-page=47>

<i><sub>a</sub></i>4 <i><sub>b</sub></i>4 <sub>2</sub>




Điều phải chứng minh.
Bài 2:


<b>Giải</b>


Ta có với mọi số nguyên dơng k:


1 1 1 1


( )


1
(<i>k</i> 1) <i>k</i>  <i>k</i> (<i>k</i> 1) <i>k</i>  <i>k</i> <i>k</i>  <i>k</i> 


= ( 1 1 )( 1 1 )


1 1


<i>k</i>


<i>k</i>  <i>k</i>  <i>k</i>  <i>k</i> 


1 1 1 1


(1 )( ) 2( )



1 1 1


<i>k</i>


<i>k</i> <i>k</i> <i>k</i> <i>k</i> <i>k</i>


    


   v× 0 1 1


<i>k</i>
<i>k</i>


 



Do đó:


1 1 1 1 1 1 1 1


... 2(1 ) 2( ) ... 2( )


2 3 2  (<i>n</i>1) <i>n</i>   2  2  3   <i>n</i>  <i>n</i>1


2(1 1 ) 2
1


<i>n</i>



  


 điều phải chứng minh.
Bài 3:


<b>Giải</b>


(1) 1 0


4
4
4
4
2
2
2
2
2
2
2






































<i>m</i> <i>mn</i> <i>n</i> <i>m</i> <i>mp</i> <i>p</i> <i>m</i> <i>mq</i> <i>q</i> <i>m</i> <i>m</i>


0


1
2
2
2
2
2
2
2
2


































 <i>m</i> <i>n</i> <i>m</i> <i>p</i> <i>m</i> <i>q</i> <i>m</i>


DÊu b»ng x¶y ra khi


</div>
<span class='text_page_counter'>(48)</span><div class='page_container' data-page=48>

<b>Gi¶i</b>

<i><sub>a</sub></i>10 <sub></sub><i><sub>b</sub></i>10



<i><sub>a</sub></i>2<sub></sub><i><sub>b</sub></i>2

 

<sub></sub><i><sub>a</sub></i>8<sub></sub><i><sub>b</sub></i>8



<i><sub>a</sub></i>4 <sub></sub><i><sub>b</sub></i>4



 <i><sub>a</sub></i>12 <i><sub>a</sub></i>10<i><sub>b</sub></i>2 <i><sub>a</sub></i>2<i><sub>b</sub></i>10 <i><sub>b</sub></i>12 <i><sub>a</sub></i>12 <i><sub>a</sub></i>8<i><sub>b</sub></i>4 <i><sub>a</sub></i>4<i><sub>b</sub></i>8 <i><sub>b</sub></i>12










 8 2

2 2

2 8

2 2

0





 <i>b</i> <i>a</i> <i>b</i> <i>b</i> <i>a</i>


<i>a</i>
<i>b</i>
<i>a</i>


 a2<sub>b</sub>2<sub>(a</sub>2<sub>-b</sub>2<sub>)(a</sub>6<sub>-b</sub>6<sub>)</sub>


 0  a2<sub>b</sub>2<sub>(a</sub>2<sub>-b</sub>2<sub>)</sub>2<sub>(a</sub>4<sub>+ a</sub>2<sub>b</sub>2<sub>+b</sub>4<sub>) </sub>


 0
Bất đẳng thức cuối đúng vậy ta có điều phải chứng minh.
Bài 5:


<b>Gi¶i</b>


Víi a > 0 , b > 0 => a + b > 0


Ta cã :


3
3


3


2



2 





 

<i>b</i> <i>a</i> <i>b</i>
<i>a</i>


<=>

2 2

2


2
2


.


2 





 





 











<i>a</i><i>b</i> <i><sub>a</sub></i> <i><sub>ab</sub></i> <i><sub>b</sub></i> <i>a</i> <i>b</i> <i>a</i> <i>b</i>


<=>


2
2


2


2 



 



 <i>ab</i> <i>b</i> <i>a</i> <i>b</i>
<i>a</i>


<=> 4a2<sub> - 4ab + 4b</sub>2 <sub> </sub><sub></sub><sub> a</sub>2<sub> + 2ab + b</sub>2



<=> 3(a2<sub> - 2ab + b</sub>2<sub> ) </sub><sub></sub><sub> 0</sub>


<=> 3(a - b)2


 0 . Bất đẳng thức này đúng


=>


3
3


3


2


2 





 

<i>b</i> <i>a</i> <i>b</i>


<i>a</i> <sub> DÊu '' = '' xảy ra khi a = b.</sub>


Bài 6:
<b>Giải</b>
Ta có:


2


b


a


b


a


a


ab


2


b


a


b


a


ab


a


b


a


a


2
2
3
2
2
2
2
2
2
2
3


















</div>
<span class='text_page_counter'>(49)</span><div class='page_container' data-page=49>

<i><b> </b></i>


2
d
c
d
c


c


2
c
b
c
b



b


2
2


3
2
2


3










<i><b> </b></i>


2
a
d
a
d


d


2


2


3






Cộng vế theo vế ta đợc:


2
d
c
b
a
a
d


d
d


c
c
c


b
b
b


a


a


2
2


3
2


2
3
2


2
3
2


2


3 <sub></sub> <sub></sub> <sub></sub>











<i><b>D</b></i><b>ạng 2 </b>–<b> Sử dụng bất đẳng thức Bunhiacopxky và bất đẳng thức phụ </b>



Bài 1:


<b>Giải</b>


ỏp dng bt ng thc Bunhiacụpxki, ta cú:




2 2 2 2 2


2 2 2 2


(1x 1y 1z) (1 1 1 )(x y z)


(x y z) 3(x y z ) (1)


      


    


Theo gi¶ thuyÕt ta cã:




2 2 2


4
x(x 1) y(y 1) z(z 1)



3
4


(x y z ) (x y z)


3


     


      


V× theo (1) 1(x y z)2 (x2 y2 z )2


3


     




2


2


1 4


(x y z) (x y z)


3 3


(x y z) 3(x y z) 4



      


  


Đặt

s x y z

Ta giải phơng trình bậc 2




2


s

3s 4 0



1 s 4


x y z 4





 

 



</div>
<span class='text_page_counter'>(50)</span><div class='page_container' data-page=50>

Bài 2:


<b>Giải</b>


Do a,b,c i xng ,gi s abc 












<i>b</i>
<i>a</i>
<i>c</i>
<i>c</i>
<i>a</i>
<i>b</i>
<i>c</i>
<i>b</i>


<i>a</i> <i>a</i> <i>b</i> <i>c</i>


2
2
2


¸p dơng BĐT Trê- b-sép ta có:






















<i>a</i> <i>b</i>


<i>c</i>
<i>c</i>
<i>a</i>
<i>b</i>
<i>c</i>
<i>b</i>
<i>a</i>
<i>c</i>
<i>b</i>
<i>a</i>
<i>b</i>
<i>a</i>
<i>c</i>
<i>c</i>
<i>c</i>
<i>a</i>


<i>b</i>
<i>b</i>
<i>c</i>
<i>b</i>
<i>a</i>
<i>a</i> .
3
.
.
.
2
2
2
2
2
2
=
2
3
.
3
1
=
2
1
Vậy
2
1
3
3

3






<i>a</i> <i>b</i>


<i>c</i>
<i>c</i>
<i>a</i>
<i>b</i>
<i>c</i>
<i>b</i>
<i>a</i>


.Dấu bằng xảy ra khi a=b=c=


3
1


Bài 3:


<b>Giải</b>


ỏp dng bất đẳng thức Bunhiacơpxki ta có:
(x2<sub> + y</sub>2<sub>)</sub>2<sub> = (</sub><i><sub>x</sub></i> <sub>1</sub> <i><sub>y</sub></i>2 <i><sub>y</sub></i> <sub>1</sub> <i><sub>x</sub></i>2






 )2 ( <i>x</i> 1<sub> ; </sub> <i>y</i> 1<sub>)</sub>


 (x2<sub> + y</sub>2<sub>)(1 - y</sub>2<sub> + 1 - x</sub>2<sub>) => x</sub>2<sub> + y</sub>2 <sub></sub><sub> 1</sub>


Ta l¹i cã : (3x + 4y)2 <sub></sub><sub> (3</sub>2<sub> + 4</sub>2<sub>)(x</sub>2<sub> + y</sub>2<sub>) </sub><sub></sub><sub> 25</sub>


=> 3x + 4y  5


Đẳng thức xảy ra <=>














4
3
0
,
0
1


2
2
<i>y</i>
<i>x</i>
<i>y</i>
<i>x</i>
<i>y</i>
<i>x</i>
<=>







5
4
5
3
<i>y</i>
<i>x</i>


Điều kiện :


2
5
2


3



<i>x</i> .


Bài 4:


<b>Giải</b>


áp dụng bất dẳng thức Bunhiacôpxki với 2 bộ 3 sè ta cã:


          <sub></sub>

 <sub></sub> <sub></sub> <sub></sub> <sub></sub> <sub></sub>








<i>b</i>.1 <i>b</i> <i>c</i>.1 <i>c</i> <i>a</i>.1 1 1 1 <i>a</i> <i>b</i> 2 <i>b</i> <i>c</i> 2 <i>c</i> <i>a</i> 2
<i>a</i>


=>

<sub></sub>

<i>a</i><i>b</i> <i>b</i><i>c</i>  <i>c</i><i>a</i>

<sub></sub>

2 3.(2<i>a</i>2<i>b</i><i>ac</i>)6


=> <i>a</i><i>b</i> <i>b</i><i>c</i> <i>c</i><i>a</i> 6


DÊu '' = '' x¶y ra khi : a = b = c =


3


1


Bài 5:


<b>Giải</b>


</div>
<span class='text_page_counter'>(51)</span><div class='page_container' data-page=51>

Ta cã <i>p</i>  <i>p a</i>  <i>p b</i>  <i>p c</i>


3 ( ) 2( ( )( ) ( )( ) ( )( ))


<i>p</i> <i>p</i> <i>a b c</i> <i>p a p b</i> <i>p b p c</i> <i>p c p a</i>


             


Kết quả này ln đúng vì <i>a b c</i>  2<i>p</i> và <i>p a o p b o p c o</i>  ,   ,   .


Theo bất đẳng thức Bunhiacơpski ta có:


1. <i>p a</i> 1. <i>p b</i> 1. <i>p c</i>  (1 1 1)(  <i>p a p b p c</i>     )  3<i>p</i>


Vậy bất đẳng thức đợc chứng minh.
Bài 6:


<b>Gi¶i</b>


áp dụng bất đẳng thức Bunhiacơpski cho các số 1,1,1, <i>a b c</i>, ,


<i>b c a</i> ta cã :


2 2 2



2 2 2 2


2 2 2


(1.<i>a</i> 1.<i>b</i> 1. )<i>c</i> (1 1 1 )(<i>a</i> <i>b</i> <i>c</i> )


<i>b</i>  <i>c</i>  <i>a</i>    <i>b</i>  <i>c</i>  <i>a</i>


2 2 2


2 2 2


1


( )( )


3


<i>a</i> <i>b</i> <i>c</i> <i>a</i> <i>b</i> <i>c</i> <i>a</i> <i>b</i> <i>c</i>


<i>b</i> <i>c</i> <i>a</i> <i>b</i> <i>c</i> <i>a</i> <i>b</i> <i>c</i> <i>a</i>


       


áp dụng bất đẳng thức Cơsi ta có:


3


3 . . 3



<i>a</i> <i>b</i> <i>c</i> <i>a b c</i>


<i>b</i>  <i>c</i>  <i>a</i>  <i>b c a</i> 


Do đó:


2 2 2


2 2 2


1


( ).3


3


<i>a</i> <i>b</i> <i>c</i> <i>a</i> <i>b</i> <i>c</i> <i>a b</i> <i>c</i>


<i>b</i> <i>c</i>  <i>a</i>  <i>b</i>  <i>c</i>  <i>a</i>  <i>b</i> <i>c</i>  <i>a</i> (đpcm).


Bài 7:


<b>Giải</b>


<i><b>T gi thit đề ra </b></i>a,b,c0


Ta cã: abbccaabc 1
c
1


b
1
a
1







 

*  1<sub>2</sub>  1<sub>2</sub>  1<sub>2</sub>  1<sub>2</sub>  1<sub>2</sub>  1<sub>2</sub>  1<sub>2</sub>  1<sub>2</sub>  1<sub>2</sub>  3


<i>a</i>
<i>a</i>
<i>c</i>
<i>c</i>


<i>c</i>
<i>b</i>
<i>b</i>


</div>
<span class='text_page_counter'>(52)</span><div class='page_container' data-page=52>

áp dụng bất đẳng thức Bunhicopsky ta có:
2
2
2
2
2


2 <sub>1</sub> <sub>1</sub> <sub>.</sub> <sub>x</sub> <sub>y</sub> <sub>z</sub>



1
z
y


x      


Do đó ta có:













b
1
b
1
a
1
3
1
b
1
b


1
a
1
2
2
2











c
1
c
1
b
1
3
1
c
1
c
1
b

1
2
2
2











a
1
a
1
c
1
3
1
a
1
a
1
c
1
2

2
2


Cộng vế theo vế ta đợc:
















c
1
b
1
a
1
3
3
1
cb
c


2
a
bc
b
2
c
ab
a
2


b2 2 2 2 2 2


3
cb
c
2
a
bc
b
2
c
ab
a
2


b2 2 2 2 2 2










<b>Dạng 3 – sử dụng Bất đẳng thức Cauchy</b>
Bài 1:
<b>Giải</b>
VT =

<i>b</i>


<i>a</i>


+

<i>a</i>


<i>b</i>


+

<i>c</i>


<i>b</i>


+

<i>b</i>


<i>c</i>


+

<i>c</i>


<i>a</i>


+

<i>a</i>


<i>c</i>


+ 3


áp dụng BĐT côsi cho các bội số :


<i>b</i>


<i>a</i>

<i>a</i>
<i>b</i>
;
<i>c</i>
<i>b</i>

<i>b</i>
<i>c</i>
;
<i>c</i>
<i>a</i>

<i>a</i>
<i>c</i>


Ta cã :


<i>b</i>
<i>a</i>


+


<i>a</i>


<i>b</i> <sub>≥ 2</sub>


<i>b</i>
<i>a</i>


<i>a</i>
<i>b</i>


. =2 ;


<i>c</i>
<i>b</i>


+


<i>b</i>
<i>c</i>


≥ 2 ;


<i>c</i>
<i>a</i>
+
<i>a</i>
<i>c</i>
≥ 2
<=>

<i>b</i>


<i>a</i>


+

<i>a</i>


<i>b</i>


+

<i>c</i>


<i>b</i>



+

<i>b</i>


<i>c</i>


+

<i>c</i>


<i>a</i>


+

<i>a</i>


<i>c</i>


≥ 6
<=>

<i>b</i>


<i>a</i>


+

<i>a</i>


<i>b</i>


+

<i>c</i>


<i>b</i>


+

<i>b</i>


<i>c</i>


+

<i>c</i>


<i>a</i>


+

<i>a</i>


<i>c</i>


+3 9



VT 9 ( đpcm ).
Bài 2:


<b>Giải</b>


</div>
<span class='text_page_counter'>(53)</span><div class='page_container' data-page=53>

a)Theo bất đẳng thức Cơsi ta có:




(p a) (p b) c


(p a)(p b)


2 2


(p a) (p c) b


(p a)(p c)


2 2


(p b) (p c) a


(p b)(p c)


2 2


  


   



  


   


  


   


Nhân theo vế ta đợc:


(p a) (p b) (p c)2 2 2 c b a. . abc


2 2 2 8


    


b)Ta áp dụng với x,y>0 ta ln có bất đẳng thức đúng:


1 1 4
x y x y
Thay các giá trị vào ta đợc:


1 1 4 4


p a p b   2p a b  c
T¬ng tù:


1 1 4 4



p b p c   2p b c  a


1 1 4 4


p a p c   2p a c  b
Céng theo vÕ ta cã:


1

1

1

2(

1 1 1

)



p a p b p c

a b c



Bài 3:


<b>Giải</b>


ỏp dng bt đẳng thức Cơsi , ta có:


1
2
2


1
)
1
(


1    


 <i>a</i> <i>a</i>



<i>a</i>


T¬ng tù : 1


2
1 
 <i>b</i>


<i>b</i> ; 1


2
1 
 <i>c</i>


<i>c</i>


</div>
<span class='text_page_counter'>(54)</span><div class='page_container' data-page=54>

5
,
3
3
2
1
1


1        
 <i>b</i> <i>c</i> <i>a</i> <i>b</i> <i>c</i>


<i>a</i>


Dấu đẳng thức xảy ra khi a = b = c =0 trái với giả thiết : a + b + c = 1


Vy : <i>a</i>1 <i>b</i>1 <i>c</i>13,5


Bài 4:


<b>Giải</b>


Theo bt ng thc Cơsi ta có:


( ) ( )


( )( )


2 2


<i>p a</i> <i>p b</i> <i>c</i>


<i>p a p b</i>      


( ) ( )


( )( )


2 2


<i>p b</i> <i>p c</i> <i>a</i>


<i>p b p c</i>      


( ) ( )



( )( )


2 2


<i>p c</i> <i>p a</i> <i>b</i>


<i>p c p a</i>      


 ( ) (2 ) (2 )2 . .


2 2 2


<i>c b a</i>
<i>p a</i> <i>p b</i> <i>p c</i> 


 ( )( )( )


8


<i>abc</i>


<i>p a p b p c</i>    . Điều phải chứng minh.


<b>Dạng 4 chứng minh bằng phản chứng</b>
Bài 1:


<b>Giải:</b>


Gi s c 3 bt ng thc trên đều đúng, có nghĩa:



1 1 1


(1 ) ; (1 ) ; (1 )


4 4 4


1


(1 )(1 )(1 ) ( , , (0;1))


64


<i>a</i> <i>b</i> <i>b</i> <i>c</i> <i>c</i> <i>a</i>


<i>abc</i> <i>a</i> <i>b</i> <i>c</i> <i>Do a b c</i>


     


     


Ta cã


2
2


2


1 1


(1 ) ( )



2 4


1 1 1


4 2 4


<i>a</i> <i>a</i> <i>a</i> <i>a</i> <i>a</i>


<i>a</i>


<sub></sub> <sub></sub> 


    <sub></sub><sub></sub>  <sub></sub>  <sub></sub>


 


 


 


 


  <sub></sub>  <sub></sub> 


 


T¬ng tù ta cã: 0 (1 ) 1
4



<i>a</i> <i>a</i>


  


</div>
<span class='text_page_counter'>(55)</span><div class='page_container' data-page=55>



1


0 (1 )


4
1


0 (1 )


4


<i>b</i> <i>b</i>


<i>c</i> <i>c</i>


  


  


1
(1 ) (1 ) (1 )


64
1


(1 )(1 )(1 )


64


<i>a</i> <i>a b</i> <i>b c</i> <i>c</i>


<i>abc</i> <i>a</i> <i>b</i> <i>c</i>


    


 


Bài 2:


<b>Giải:</b>


Chng minh bng phn chng. Gi s trong 25 số tự hiên đả cho, khơng có hai số
nào bằng nhau. Khơng mất tính tổng qt, giả sử


<i>a</i>

<sub>1</sub>

<i>a</i>

<sub>2</sub>

 

...

<i>a</i>

<sub>25</sub>


Suy ra

<i>a</i>

<sub>1</sub>

1,

<i>a</i>

<sub>2</sub>

2,...,

<i>a</i>

<sub>25</sub>

25



ThÕ th×


1 2 25


1

1

<sub>...</sub>

1



<i>a</i>

<i>a</i>

 

<i>a</i>

1

<sub>1</sub>

1

<sub>2</sub>

 

...

1

<sub>25</sub>

(1)


Ta l¹i cã


1

1

...

1



1

2

 

25

2 25 1 9

 

(2)


Tõ (1) vµ (2) suy ra


1 2 25


1

1

<sub>...</sub>

1



<i>a</i>

<i>a</i>

 

<i>a</i>

< 9, trái với giả thiết. Vậy tồn tại


hai sè b»ng nhau trong 25 sè

<i>a a</i>

<sub>1 2</sub>

, ,...,

<i>a</i>

<sub>25</sub>.


<b>D¹ng 5 - Phơng pháp lợng giác</b>


Bài 1:


<b>Giải</b>


</div>
<span class='text_page_counter'>(56)</span><div class='page_container' data-page=56>

Đặt






























cossin


32


cos


sinA


cos2


b


sin1


a


cos2


b


sin1



a

<sub>2</sub>

<sub>2</sub>



A ) 2


6
2
sin(
2
2


cos
2
1
2
sin
2
3
2
2
cos
2
sin


3        


(đpcm)


Bài 2:


<b>Giải:</b>


Bin i bt ng thc: a2<sub> + b</sub>2<sub> + 2(b-a)  - 1  (a-1)</sub>2<sub> + (b + 1)</sub>2<sub> 1</sub>


Đặt











cos


R


1


b


sin


R


1


a



với R 0

a(

)1

2

b(

)1

2

R

2



1


cos


R


b


1


sinR


a




















Ta cã: 5a12b7 13 5(Rsin1)12(Rcos1)7 13


 R


13
5
arccos
sin
R
cos
13
12
sin
13
5
R
1
13
cos
R
12
sin
R


5  





















Từ đó  (a-1)2<sub> + (b+1)</sub>2<sub> = R</sub>2<sub>  1  a</sub>2<sub> + b</sub>2<sub> + 2(b - a) - 1 (pcm)</sub>


Bài 3:


<b>Giải:</b>


Từ đk 1 - a2<sub>  0  |a|  1 nªn</sub>


Đặt a = cos với 0      <sub>1 </sub> <sub>a</sub>2 <sub> = sin. Khi đó ta có:</sub>


A=2 3a22a 1 a2 2 3cos22cossin 3(1cos2)sin2



= 3
3
2
sin
2
3
2
sin
2
1
2
cos
2
3


2 






 














  3 2A 32(đpcm)


Bài 4:


<b>Giải:</b>


Do |a| 1 nên :


</div>
<span class='text_page_counter'>(57)</span><div class='page_container' data-page=57>

Đặt a =



cos


1


với














2
3
,
2
;


0 a21 tg2 tg<sub>. Khi đó:</sub>


A = 2


3
sin
2
cos
3
sin
cos
)
3
tg
(
a
3
1
a2







 













(đpcm)
Bài 5:
<b>Giải:</b>


Do |a| 1 nên:


Đặt a =



cos


1



với  





 






 
2
3
,
2
;


0  a21 tg2 tg. Khi đó:


A =


2
2
a
1
a
12



5  <sub>= (5-12tg)cos</sub>2<sub> = 5cos</sub>2<sub>-12sincos=</sub>   <sub></sub> <sub>6</sub><sub>sin</sub><sub>2</sub><sub></sub>


2
)
2
cos
1
(
5
=
















13
5
arccos


2
cos
2
13
2
5
2
sin
13
12
2
cos
13
5
2
13
2
5




- 4 = .1 9


2
13
2
5
13
5


arccos
2
cos
2
13
2
5
A
)
1
(
2
13
2
5

















(đpcm)


Bài 6:


<b>Gi¶i:</b>


Đặt a = tg, b = tg, c = tg. Khi đó bất đẳng thức 



)
tg
1
)(
tg
1
(
|
tg
tg
|
)
tg
1
)(
tg
1
(
|
tg


tg
|
)
tg
1
)(
tg
1
(
|
tg
tg
|
2
2
2
2
2
2

















































cos
.
cos
)
sin(
.
cos
cos
cos
.
cos
)
sin(
.
cos
cos
cos
.
cos
)
sin(
.
cos
cos


 sin(-)+sin(-)  sin(-). Biến đổi biểu thức vế phải ta có:


sin(-)= sin[(-)+(-)] = sin(-)cos(-)+sin(-)cos(-) 



sin(-)cos(-)+sin(-)cos(-)=sin(-)cos(-)+sin(-)cos(-)


 sin(-).1 + sin(-).1 = sin(-) + sin(-)  (đpcm)
Bài 7:


</div>
<span class='text_page_counter'>(58)</span><div class='page_container' data-page=58>

(1) 1
d
b
1
a
c
1
ab
cd
d
b
1
a
c
1
1
1
)
d


b
)(
c
a
(
cd
)
d
b
)(
c
a
(
ab







































Đặt tg2<sub>=</sub>


a
c


, tg2<sub>=</sub>


b


d


víi ,  




 
2
,


0 <sub>  Biến đổi bất đẳng thức</sub>


 cos cos sin sin 1


)
tg
1
)(
tg
1
(
tg
.
tg
)
tg
1
)(
tg


1
(


1 2 2 2 2


2
2


2
2
2


2     














 cos cos + sin sin = cos(-)  1 đúng  (đpcm)


DÊu b»ng x¶y ra  cos(-) = 1  =  <sub>a</sub>c <sub>b</sub>d <sub>.</sub>



Bài 8:


<b>Giải:</b>


t a = tg, b = tg. Khi đó


)
tg
)(
tg
(
)
tg
tg
)(
tg
tg
(
)
b
)(
a
(
)
ab
)(
b
a
(
















2
2
2


2 <sub>1</sub> <sub>1</sub>


1
1
1
1
=















cos
.
cos
sin
.
sin
cos
.
cos
.
cos
.
cos
)
sin(
.
cos
cos2 2


=


2

1
2
2
1











 )cos( ) sin ( )


sin( (đpcm).


<b>Dạng 6 </b><b> Phơng pháp qui nạp</b>


Bài 1:


<b>Giải:</b>


Với n =2 ta có


2
1
2


4
1


1 (đúng)


Giả sử BĐT (1) đúng với n =k ta phải chứng minh
BĐT (1) đúng với n = k+1


ThËt vËy khi n =k+1 th×:


(1) 


1
1
2
)
1
(
1
1
....
2
1
1
1
2
2
2


2   <i><sub>k</sub></i> <i><sub>k</sub></i><sub></sub> <i><sub>k</sub></i><sub></sub>



Theo giả thiết quy nạp:




1


1
2
1
1
1
2
)
1
(
1
1
....
2
1
1
1
2
2
2
2


2   <sub></sub>












<i>k</i>
<i>k</i>
<i>k</i>
<i>k</i>
<i>k</i>


</div>
<span class='text_page_counter'>(59)</span><div class='page_container' data-page=59>



<i>k</i>  <i>k</i>
<i>k</i>
<i>k</i>
1
1
1
1
1
)
1
(
1
....


1
1
2
2
2 








 2 ( 2) ( 1)2


1
)
1
(
1
1








<i>k</i>


<i>k</i>
<i>k</i>
<i>k</i>
<i>k</i>
<i>k</i>


 k2+2k<k2+2k+1


Điều này đúng .Vậy bất đẳng thức (1) đợc chứng minh
Bài 2:


<b>Gi¶i:</b>


Ta thấy BĐT (1) đúng với n= 1


Giả sử BĐT (1) đúng với n=k ta phải chứng minh BĐT đúng với n=k+1
Thật vậy với n = k+1 ta có


(1) 


1
2







<i>a</i><i>b</i> <i>k</i>





2


1


1 


 <sub></sub> <i>k</i>
<i>k</i> <i><sub>b</sub></i>
<i>a</i>

2
.
2
<i>b</i>
<i>a</i>
<i>b</i>


<i>a</i> <i>k</i> 







 

2


1
1 


 <sub></sub> <i>k</i>
<i>k</i> <i><sub>b</sub></i>


<i>a</i> <sub> (2)</sub>


 VÕ tr¸i (2) 


2
4
2
.
2
1
1
1


1   












 <i>k</i> <i>k</i> <i>k</i> <i>k</i> <i>k</i> <i>k</i> <i>k</i>


<i>k</i> <i><sub>b</sub></i> <i><sub>a</sub></i> <i><sub>b</sub></i> <i><sub>a</sub></i> <i><sub>ab</sub></i> <i><sub>a</sub></i> <i><sub>b</sub></i> <i><sub>b</sub></i> <i><sub>a</sub></i> <i><sub>b</sub></i>


<i>a</i>
 0
4
2
1
1
1
1





   


 <i>k</i> <i>k</i> <i>k</i> <i>k</i> <i>k</i>


<i>k</i> <i><sub>b</sub></i> <i><sub>a</sub></i> <i><sub>ab</sub></i> <i><sub>a</sub></i> <i><sub>b</sub></i> <i><sub>b</sub></i>


<i>a</i>


<i>ak</i>  <i>bk</i>

.<i>a</i> <i>b</i>0 (3)


Ta chøng minh (3)


(+) Gi¶ sư a  b và giả thiết cho a -b a  <i>b</i>



 <i><sub>a</sub>k</i> <i><sub>b</sub>k</i> <i><sub>b</sub>k</i>




 

(a

k

b )(a b) 0

k



(+) Giả sử a < b và theo gi¶ thiÕt a < b  <i>k</i> <i>k</i> <i>k</i> <i>k</i>


<i>b</i>
<i>a</i>
<i>b</i>


<i>a</i>   


<i>ak</i>  <i>bk</i>

.<i>a</i> <i>b</i>0


Vậy BĐT (3)ln đúng ta có (đpcm)
Bài 3:


<b>Gi¶i:</b>


Víi n = 1 ta cã <i><sub>a</sub></i>2 <i><sub>b</sub></i>2 <i><sub>c</sub></i>2


  (định lý Pitago).


VËy víi n = 1 <i><sub>a</sub></i>2 <i><sub>b</sub></i>2 <i><sub>c</sub></i>2


  đúng



Ta giả sử BĐT đúng với n= k nghĩa là ta có:
<i><sub>a</sub></i>2<i>k</i> <i><sub>b</sub></i>2<i>k</i> <i><sub>c</sub></i>2<i>k</i>


</div>
<span class='text_page_counter'>(60)</span><div class='page_container' data-page=60>

xÐt víi n = k +1 ta cã:


2(<i>k</i> 1) 2(<i>k</i> 1) <sub>(</sub> 2<i>k</i> 2<i>k</i><sub>)(</sub> 2 2<sub>)</sub> 2 2<i>k</i> 2 2<i>k</i> 2<i>k</i><sub>.</sub> 2 2(<i>k</i> 1)


<i>a</i>  <i>b</i>  <i>a</i> <i>b</i> <i>a</i> <i>b</i> <i>a a</i> <i>b b</i> <i>c c</i> <i>c</i> 


       


Vậy BĐT đúng với n = k +1. iu phi chng minh.


<b>Dạng 7 - Phơng pháp áp dơng c¸c tÝnh chÊt cđa c¸c d·y tØ sè b»ng nhau</b>
<b>Giải</b>


Ta biết với a, b, c dơng và <i>a</i> 1 <i>a</i> <i>a c</i>


<i>b</i> <i>b</i> <i>b c</i>




  




V× <i>a</i> <i>a c</i>


<i>b</i> <i>b c</i>








1



<i>ab ac ab bc</i>


<i>a</i>

<i>c</i>


<i>ac bc</i>



<i>b</i>

<i>c</i>







VËy víi a, b, c, d > o th×:


<i>a</i> 1


<i>a b c</i>   , 1 , 1 , 1


<i>b</i> <i>c</i> <i>d</i>


<i>b c d</i>   <i>c d a</i>   <i>d a b</i>  


Do đó ta đợc:


<i>a</i> <i>a</i> <i>a d</i>



<i>a b c d</i> <i>a b c</i> <i>a b c d</i>




 


       


<i>b</i>
<i>a b c d</i>  


<i>b</i> <i>b a</i>


<i>b c d</i> <i>b c d a</i>




 


    


<i>c</i> <i>c</i> <i>c b</i>


<i>a b c d</i> <i>c d a</i> <i>c d a b</i>




 



       


<i>d</i> <i>d</i> <i>d c</i>


<i>a b c d</i> <i>d a b</i> <i>d a b c</i>




 


       


Cộng từng vế bốn đẳng thức trên ta đợc:


1 <i>a</i> <i>b</i> <i>c</i> <i>d</i> 2


<i>a b c b c d</i> <i>c d a</i> <i>d a b</i>


    


     (đpcm).


Bài 2:


<b>Giải:</b>


</div>
<span class='text_page_counter'>(61)</span><div class='page_container' data-page=61>

Không mất tính tổng quát ta giả sử :


<i>c</i>
<i>a</i>



<i>d</i>
<i>b</i>


Từ :


<i>c</i>
<i>a</i>


<i>d</i>
<i>b</i>




<i>d</i>
<i>b</i>
<i>d</i>
<i>c</i>


<i>b</i>
<i>a</i>
<i>c</i>
<i>a</i>










1


<i>c</i>
<i>a</i>


v× a + b = c + d


NÕu: b 998 th×


<i>d</i>
<i>b</i>


998


 


<i>d</i>
<i>b</i>
<i>c</i>
<i>a</i>


  999


NÕu: b = 998 th× a = 1
<i>d</i>
<i>b</i>
<i>c</i>
<i>a</i>



=


<i>d</i>
<i>c</i>


999
1


Đạt giá trị lớn nhất khi d = 1; c = 999


Vậy giá trị lớn nhất cña


<i>d</i>
<i>b</i>
<i>c</i>
<i>a</i>


 = 999 +
999


1


khi a = d =1; c = b = 999.


Bài 3:


<b>Giải:</b>


Từ



<i>b</i>
<i>a</i>


<


<i>d</i>
<i>c</i>


2
2 <i><sub>d</sub></i>


<i>cd</i>
<i>b</i>
<i>ab</i>






<i>d</i>
<i>c</i>
<i>d</i>
<i>cd</i>
<i>d</i>
<i>b</i>


<i>cd</i>
<i>ab</i>
<i>b</i>


<i>ab</i>







 <sub>2</sub> <sub>2</sub> <sub>2</sub>


2


VËy


<i>b</i>
<i>a</i>


<


<i>d</i>
<i>c</i>
<i>d</i>
<i>b</i>


<i>cd</i>
<i>ab</i>







2


2 điều phải chứng minh.


<b>Dạng 8 </b><b> Phơng pháp dùng tam thøc bËc hai</b>


<b>Bµi 1: Gi¶i:</b>


Ta cã: (1) a22(b 3c d)a (b c d)     2 8bd 0 (2)


XÐt tam thøc f(a) a 22(b 3c d)a (b c d)     2 8bd


Tam thøc f(a) cã:


2 2


(b 3c d) (b c d) 8bd


8(c b)(c d)
D


D


       


   


Theo gi¶ thiÕt ta cã: <sub>D </sub> <sub>0</sub>
Suy ra: f(a)>0



Suy ra (2) đúng vậy nên (1) cũng đúng.
Bài 2:


<b>Gi¶i:</b>


Theo gi¶ thiÕt ta cã: (p2 a2 b )(q2 2 c2 d ) 02 


Ta cã thĨ gi¶ sư: (p2 a2 b ) 02  Vµ p 0


</div>
<span class='text_page_counter'>(62)</span><div class='page_container' data-page=62>

Ta cã:f(x) (px q) 2 (ax c) 2 (bx d) 2


Đặt: 0


p
x


q


<sub> Ta cã: </sub> 2 2


0 0 0


f(x ) (ax  c)  (bx  d) 0


Suy ra <sub>D </sub><sub>0</sub>, tc l (1) ỳng.


<b>Dạng 9 </b><b> Phơng pháp dùng tính chất bắc cầu</b>


Bài 1:



<b>Giải</b>


Ta có: (1 <i>a</i>)(1 <i>b</i>) 1  <i>a b ab</i>   1 <i>a b</i> (1)


Vì 1 <i>c</i>0 nên:


(1 )(1 )(1 ) (1 )(1 ) (2)


(1 )(1 ) 1 ( ) 1 (3)


<i>a</i> <i>b</i> <i>c</i> <i>a b</i> <i>c</i>


<i>a b</i> <i>c</i> <i>a b c c a b</i> <i>a b c</i>


      


            


Tõ (2) vµ (3) ta suy ra:(1 <i>a</i>)(1 <i>b</i>)(1 <i>c</i>) 1  <i>a b c</i> 


VËy: (1 <i>a</i>)(1 <i>b</i>)(1 <i>c</i>)(1 <i>d</i>) (1  <i>a b c</i>  )(1 <i>d</i>) 1  <i>a b c d</i>  


V× <i>d a b c</i>( ) 0


Bài 2:


<b>Giải</b>


Vì <i>a b c</i>  3 nªn cã Ýt nhÊt mét trong 3 bộ số a, b, c không nhỏ hơn 1, giả sử



1


<i>a </i>


Vì 1 <i>a</i> 2 nên (<i>a</i> 1)(<i>a</i> 2)<i>a</i>2 3<i>a</i>  2 0 <i>a</i>(3 <i>a</i>) 2


Suy ra <i>ab bc ca a b c</i>   (  )<i>bc a</i> (3 <i>a</i>)<i>bc</i>2 (1)


VËy


2 2 2 <sub>(</sub> <sub>)</sub>2 <sub>2(</sub> <sub>)</sub>


9 2( ) 5


<i>a</i> <i>b</i> <i>c</i> <i>a b c</i> <i>ab bc ca</i>


<i>ab bc ca</i>


       


     theo (1).


Bµi 3:


<b>Giải</b>


áp dụng công thức Hê - rông về tính diƯn tÝch tam gi¸c:


( )( )( )



<i>S</i>  <i>p p a p b p c</i>   Víi 1


2 2


<i>a b c</i>


<i>p</i>    theo gi¶ thiÕt.


Do đó: 2 1 1( )(1 )(1 )


2 2 2 2


<i>S</i>   <i>a</i>  <i>b</i>  <i>c</i>


</div>
<span class='text_page_counter'>(63)</span><div class='page_container' data-page=63>

2


16 (1 2 )(1 2 )(1 2 )


1 2 2 2 4 4 4 8


1 4( ) 8 0


<i>S</i> <i>a</i> <i>b</i> <i>c</i>


<i>a</i> <i>b</i> <i>c</i> <i>ab</i> <i>ac</i> <i>bc</i> <i>abc</i>


<i>ab bc ac</i> <i>abc</i>


   



       


     


Suy ra: 4 1 2 2 2


2


<i>abc</i>  <i>ab</i> <i>bc</i> <i>ca</i>


Mµ 2ab+2bc+2ca=(<i>a b c</i>  )2  (<i>a</i>2<i>b</i>2 <i>c</i>2) 1 (  <i>a</i>2 <i>b</i>2<i>c</i>2)


Nªn 4 1 1 2 2 2 2 2 2 4 1


2 2


<i>abc</i>   <i>a</i>  <i>b</i>  <i>c</i>  <i>a</i> <i>b</i> <i>c</i> <i>abc</i>


Bài 4:


<b>Giải</b>


Nếu <i>a </i>1 thì tõ <i>b c a</i>  1 suy ra a+b+c >2, vô lý. Vậy 0 < a < 1.
Tơng tự ta cã: 0 < b < 1 vµ 0 < c < 1.


Ta cã: (1 <i>a</i>)(1 <i>b</i>)(1 <i>c</i>) 1  <i>a b c ab bc ca abc</i>      0 suy ra


1


<i>abc ab bc ca</i>    vì <i>a b c</i> 2 (1)



Mà 4 ( <i>a b c</i>  )2 <i>a</i>2 <i>b</i>2<i>c</i>2 2(<i>ab bc ca</i>  ), <i>suy ra</i>:


2 2 2


1


2 ( )


2


<i>ab bc ca</i>    <i>a</i> <i>b</i> <i>c</i> (2)


Tõ (1) vµ (2) ta suy ra:


2 2 2 2 2 2


1


1 ( ) 2 2


2


<i>abc</i>  <i>a</i> <i>b</i> <i>c</i>  <i>a</i> <i>b</i> <i>c</i>  <i>abc</i>


<b>Dạng 10 - Phơng phỏp dựng cỏc bt ng thc trong tam giỏc</b>


Bài 1:


<b>Giải</b>



Ta cã: <i>a b</i>  (<i>a b c</i>  )2 (<i>b b c</i>  )2 (2<i>b c</i> )2


Ta chøng tá


2 2 2


(2 ) 9 4 5 0


( )(4 ) 0


<i>b c</i> <i>bc</i> <i>b</i> <i>bc c</i>


<i>b c</i> <i>b c</i>


     


   


Kết quả trên luôn đúng vì <i>b c</i>  <i>b c</i> 0 và 4<i>b c</i> 2<i>b b a c</i>   0 <i>do b a</i>


vµ <i>c a b</i> .


</div>
<span class='text_page_counter'>(64)</span><div class='page_container' data-page=64>

Bài 2:


<b>Giải</b>


Vỡ a, b, c là độ dài 3 cạnh của tam giác nên:





















<i>b</i>
<i>a</i>
<i>c</i>


<i>c</i>
<i>a</i>
<i>b</i>


<i>c</i>
<i>b</i>
<i>a</i>


0


0
0




0
0
0


<i>a b c P</i>
<i>a c b Q</i>
<i>b c a R</i>


   





 <sub></sub>    


    


Theo bất đẳng thức Cosi ta có:


2


2


2



<i>P</i> <i>Q</i>


<i>PQ</i>


<i>P</i> <i>R</i>


<i>PR</i>


<i>R</i> <i>Q</i>


<i>QR</i>














2 2 2


. .


2 2 2



<i>P Q P R R Q</i>


<i>P Q R</i> <i>PQR</i>


  


  


( )( )( )


<i>bca</i> <i>a b c b c a a c b</i>


        Điều phải chứng minh.


Bài 3:


<b>Giải</b>


Đặt <i>f p</i>

<sub> </sub>

<i>pa</i>2 <i>qb</i>2  <i>pqc</i>2 v× p + q = 1  <i>q</i>  1 <i>p</i>


2 2 2


2 2 2 2 2 2


( ) (1 ) (1 )


( )


<i>f p</i> <i>pa</i> <i>p b</i> <i>p</i> <i>p c</i>



<i>c p</i> <i>a</i> <i>b</i> <i>c p b</i>


     


    


Lµ mét tam thøc bËc hai theo p cã biÖt sè




2 2 2 2 2 2


2 2 2 2 2 2


2 2 2 2


( ) 4


( 2 )( 2 )


( ) ( )


( )( )( )


<i>a</i> <i>b</i> <i>c</i> <i>b c</i>


<i>a</i> <i>b</i> <i>c</i> <i>bc a</i> <i>b</i> <i>c</i> <i>bc</i>


<i>a</i> <i>b c</i> <i>a</i> <i>b c</i>



<i>a b c a b c a b c a b c</i>


    


      


   


<sub></sub>   <sub> </sub>   <sub></sub>


        


Trong mét tam gi¸c ta cã:


</div>
<span class='text_page_counter'>(65)</span><div class='page_container' data-page=65>




















<i>b</i>
<i>a</i>
<i>c</i>


<i>c</i>
<i>a</i>
<i>b</i>


<i>c</i>
<i>b</i>
<i>a</i>


0
0


0 0


0
0


<i>a b c</i>
<i>a b c</i>
<i>a b c</i>


  






 <sub></sub>   


   


<i>a b c a b c a b c a b c</i> 

(   )(   )(   ) <0
 <sub></sub>< 0


 <i>f p</i>( ) 0 <i>p</i>


 <i>pa</i>2 <i>qb</i>2  <i>pqc</i>2 víi mäi p, q tho¶ m·n: p + q = 1.


<b>Gi¶i</b>


Ta cã:


3<sub>(</sub> 2 2<sub>)</sub> 3<sub>(</sub> 2 2<sub>)</sub> 3<sub>(</sub> 2 2<sub>) 0</sub>


<i>a b</i>  <i>c</i> <i>b c</i>  <i>a</i> <i>c a</i>  <i>b</i>  (1)


3 2 2 2 3 3


3 2 2 2 2 2


2 2 2 2 2


2 2



( )( ) ( ) ( ) 0


( ) ( ) ( ) 0


( ) ( ) ( ) ( ) 0


( )( ) ( ) ( ) 0


<i>a b c b c</i> <i>b c b c</i> <i>a b</i> <i>c</i>
<i>b c a b c</i> <i>b c</i> <i>a b</i> <i>bc c</i>


<i>b c a b a c</i> <i>a c a c</i> <i>b a</i> <i>c</i>


<i>b c a c ab a b</i> <i>c a</i> <i>b</i>


       


 


  <sub></sub>      <sub></sub> 


 


  <sub></sub>      <sub></sub> 


 


   <sub></sub>    <sub></sub> 


(<i>a b b c a c ab ca cb</i>)( )( )( ) 0 (2)



      


Kết quả (2) luôn đúng với 0 < a < b < c. Nên bất đẳng thức (1) đợc chứng minh.


<b>Gi¶i</b>


Ta cã:


2 2 2 2 2 2


(1) [( ) ] [( ) ] ( ) 0


( )( ) ( )( ) ( )( ) 0


<i>a b c</i> <i>a</i> <i>b c a</i> <i>b</i> <i>c a b</i> <i>c</i>


<i>a b c a b c a</i> <i>b c a b c a b</i> <i>c a b c a b c</i>


 


       <sub></sub>   <sub></sub> 


               




2 2 2


2 2



( ) ( ) ( ) ( ) 0


( ) 2 0


( ) ( ) 0 (2)


<i>a b c a b c a</i> <i>b a b c</i> <i>c a b c</i>


<i>a b c c</i> <i>a</i> <i>b</i> <i>ab</i>


<i>a b c c</i> <i>a b</i>


           


 


   <sub></sub>    <sub></sub> 


 


   <sub></sub> <sub></sub>


Vì a, b, c là 3 cạnh của một tam giác nên


2 2


0


( ) 0



<i>a b c</i> <i>a b c</i>


<i>c</i> <i>a b</i> <i>c</i> <i>a b</i>


    


 




 


  <sub></sub>   


</div>
<span class='text_page_counter'>(66)</span><div class='page_container' data-page=66>

Vậy (2) đúng nên BĐT (1) đợc chứng minh.


<b>Dng 11 </b><b> Phng phỏp i bin s</b>


Bài 1


<b>Giải:</b>


Đặt : a =


)
1
)(
1



( 2 2


2
2
<i>y</i>
<i>x</i>
<i>y</i>
<i>x</i>




vµ b =


)
1
)(
1
(
1
2
2
2
2
<i>y</i>
<i>x</i>
<i>y</i>
<i>x</i>





=> ab = <sub>2</sub> <sub>2</sub> <sub>2</sub> <sub>2</sub>


2
2
2
2
)
1
(
)
1
(
)
1
)(
(
<i>y</i>
<i>x</i>
<i>y</i>
<i>x</i>
<i>y</i>
<i>x</i>





Ta cã dƠ thÊy víi mäi a, b th× : - 2 <sub>(</sub> <sub>)</sub>2



4
1
)
(
4
1
<i>b</i>
<i>a</i>
<i>ab</i>
<i>b</i>


<i>a</i>   


Mµ : (a - b)2<sub> = </sub>


2
2 <sub>1</sub>
2
1 <sub></sub>







<i>x</i>


(a + b)2<sub> = </sub>



2
2 <sub>1</sub>
2
1 <sub></sub>







<i>y</i>


Suy ra : -


4
1


 ab 


4
1


.


Bµi 2:


<b>Gi¶i</b>



Ta cã : <i><sub>x</sub></i> <i><sub>y</sub></i> <i><sub>z</sub></i>




2
1
=
)
(
)
(
1
<i>z</i>
<i>x</i>
<i>y</i>


<i>x</i>  


≤ <sub>4</sub>1 (<i><sub>x </sub></i>1 <i><sub>y</sub></i> + <i><sub>y </sub></i>1 <i><sub>z</sub></i> ) ≤ <sub>16</sub>1 ( 1<i><sub>x</sub></i> + 1<i><sub>y</sub></i> + 1<i><sub>z</sub></i> + 1<i><sub>z</sub></i> )


T¬ng tù:


<i>z</i>
<i>y</i>
<i>x</i>2 


1

16
1


(
<i>x</i>
1


+ 1<i><sub>y</sub></i> +


<i>z</i>
1
+
<i>z</i>
1
)
<i>z</i>
<i>y</i>
<i>x</i> 2
1


 ≤ <sub>16</sub>


1


(


<i>x</i>


1


+ 1<i><sub>y</sub></i> +



<i>z</i>
1
+
<i>z</i>
1
)


Cộng theo vế 3 BĐT trên:


<i>z</i>
<i>y</i>
<i>x</i>
2


1


+ <i><sub>x</sub></i><sub></sub><sub>2</sub>1<i><sub>y</sub></i><sub></sub><i><sub>z</sub></i> + <i><sub>x</sub></i><sub></sub><i><sub>y</sub></i>1<sub></sub><sub>2</sub><i><sub>z</sub></i> ≤


16
1


. 4 ( 


<i>x</i>
1
<i>y</i>
1
+
<i>z</i>
1


)


Mµ 


<i>x</i>
1
<i>y</i>
1
+
<i>z</i>
1
= 4


</div>
<span class='text_page_counter'>(67)</span><div class='page_container' data-page=67>

VËy <i><sub>x</sub></i> <i><sub>y</sub></i> <i><sub>z</sub></i>




2


1


+ <i><sub>x</sub></i> <i><sub>y</sub></i> <i><sub>z</sub></i>



2


1


+ <i><sub>x</sub></i> <i><sub>y</sub></i>1 <sub>2</sub><i><sub>z</sub></i>





 ≤ 1


DÊu “=” xảy ra khi x = y = z =


3
4


.


Bài 3:


<b>Gi¶i</b>


Ta đặt


1



1



1



<i>a</i>


<i>x</i>



<i>b</i>


<i>y</i>



<i>c</i>



<i>z</i>


















víi

<i>x y z </i>

, ,

0

v do à

<i>abc </i>

1

nªn <i>xyz </i>1


Nªn B§T


2 2 2 <sub>3</sub>


2


<i>x</i> <i>y</i> <i>z</i>


<i>y z</i> <i>z x</i> <i>x y</i>







Mặt khác theo BĐT Cauchy- Schwarz ta cã:


 

 



2 2 2


2


<i>x</i> <i>y</i> <i>z</i>


<i>y z</i> <i>z x</i> <i>x y</i> <i>x y z</i>


<i>y z</i> <i>z x</i> <i>x y</i>


 


      <sub></sub>   <sub></sub>  


  <sub></sub> <sub></sub> <sub></sub>


 


2 2 2 <sub>3</sub>3 <sub>3</sub>


2 2 2


<i>xyz</i>



<i>x</i> <i>y</i> <i>z</i> <i>x y z</i>


<i>y z</i> <i>z x</i> <i>x y</i>


   


 <sub></sub>   <sub></sub>  


  


 


Vậy BĐT đợc chứng minh.
Dấu “=” xảy ra  <i>a b c</i>  1


Bµi 4:


</div>
<span class='text_page_counter'>(68)</span><div class='page_container' data-page=68>

Từ giả thiết ta có thể đặt:


<i>a</i>
<i>x</i>


<i>a b c</i>
<i>b</i>
<i>y</i>


<i>a b c</i>
<i>c</i>
<i>z</i>



<i>a b c</i>




 <sub> </sub>








 





 <sub> </sub>




với a,b,c >0


Nên BĐT CM <i>a b c</i> 4.<i>a b c</i> 9.<i>a b c</i> 36


<i>a</i> <i>b</i> <i>c</i>



     


  


<i>b</i> <i>c</i> 4.<i>a</i> 4.<i>c</i> 9.<i>a</i> 9.<i>b</i> 22


<i>a a</i> <i>b</i> <i>b</i> <i>c</i> <i>c</i>


      


4. 9. 4. 9. 2 .4. 2 .9. 2 4. .9. 22


<i>b</i> <i>a</i> <i>c</i> <i>a</i> <i>c</i> <i>b</i> <i>b</i> <i>a</i> <i>c</i> <i>a</i> <i>c</i> <i>b</i>


<i>a</i> <i>b</i> <i>a</i> <i>c</i> <i>b</i> <i>c</i> <i>a</i> <i>b</i> <i>a</i> <i>c</i> <i>b</i> <i>c</i>


     


 <sub></sub>  <sub></sub><sub></sub>  <sub></sub><sub></sub>  <sub></sub>   


      (đúng)


DÊu “=” x¶y ra


1


6



2

<sub>1</sub>



3

3




1


2



<i>x</i>



<i>b</i>

<i>a</i>



<i>y</i>



<i>c</i>

<i>a</i>



<i>z</i>











<sub></sub>

<sub></sub>





<sub></sub>









Bài 5:


<b>Giải</b>


T 2 1 1 1 1


1 1 1


<i>xyz</i> <i>x y z</i>


<i>x</i> <i>y</i> <i>z</i>


       


  


Ta đặt 1 , 1 , 1


1<i>x</i> <i>a</i> 1<i>y</i> <i>b</i> 1<i>z</i> <i>c</i> với <i>a b c </i>, , 0


1 1 1


, ,


<i>a</i> <i>b c</i> <i>b</i> <i>a c</i> <i>c</i> <i>a b</i>


<i>x</i> <i>y</i> <i>z</i>



<i>a</i> <i>a</i> <i>b</i> <i>b</i> <i>c</i> <i>c</i>


     


      


</div>
<span class='text_page_counter'>(69)</span><div class='page_container' data-page=69>

Nên BĐT cần CM  CM BĐT <sub>.</sub> <sub>.</sub> <sub>.</sub> 3


2


<i>a</i> <i>b</i> <i>b</i> <i>c</i> <i>c</i> <i>a</i>


<i>b c c a</i>   <i>c a a b</i>   <i>a b b c</i>  


Mặt khác ta có: . 1


2


<i>a</i> <i>b</i> <i>a</i> <i>b</i>


<i>b c c a</i> <i>a c b c</i>


 


 <sub></sub>  <sub></sub>


     


. 1


2


<i>b</i> <i>c</i> <i>b</i> <i>c</i>


<i>c a a b</i> <i>b a c a</i>


 


 <sub></sub>  <sub></sub>


  <sub></sub>   <sub></sub>


. 1
2


<i>c</i> <i>a</i> <i>c</i> <i>a</i>


<i>a b b c</i> <i>c b a b</i>


 


 <sub></sub>  <sub></sub>


     


Nên


1 3


. . .



2 2


<i>a</i> <i>b</i> <i>b</i> <i>c</i> <i>c</i> <i>a</i> <i>a</i> <i>b</i> <i>b</i> <i>c</i> <i>c</i> <i>a</i>


<i>b c c a</i> <i>c a a b</i> <i>a b b c</i> <i>a c b c b a c a c b a b</i>


 


   <sub></sub>      <sub></sub>


             


Vậy BĐT luôn đúng


Dấu “=” xảy ra  <i>x</i>  <i>y z</i> 2.


<b>Dạng 12 </b>–<b> Phơng pháp làm trội (chứng minh bất đẳng thức có n số hạng).</b>
<b>H</b>


<b> íng dÉn:</b>


Bµi 1:


a. Ta cã:


 



2 1

(2 1)



1 1 1 1 1


.


2 1 . 2 1 2 (2 1).(2 1) 2 2 1 2 1


<i>k</i> <i>k</i>


<i>n</i> <i>n</i> <i>k</i> <i>k</i> <i>k</i> <i>k</i>


   <sub></sub> <sub></sub>


  <sub></sub>  <sub></sub>


    <sub></sub>   <sub></sub>


Cho n chạy từ 1 đến k .Sau đó cộng lại ta có


1 1 1 1 2 1


... . 1


1.3 3.5 (2<i>n</i> 1).(2<i>n</i> 1) 2 2<i>n</i> 1 2


 


    <sub></sub>  <sub></sub>


    



b. Ta cã:




1 1 1 1 1 1


1 ... 1 ...


1.2 1.2.3 1.2.3...<i>n</i> 1.2 1.2.3 <i>n</i> 1 .<i>n</i>


        




1 1 1 1 1 1


1 1 .... 2 2


2 2 3 <i>n</i> 1 <i>n</i> <i>n</i>


     


<sub></sub>  <sub> </sub>  <sub></sub> <sub></sub>  <sub></sub>  


     


</div>
<span class='text_page_counter'>(70)</span><div class='page_container' data-page=70>

<b>I - Tìm GTLN </b><b> GTNN</b>


<b>Giải</b>



Vì x,y,z > 0 ,áp dụng BĐT Côsi ta có:
x+ y + z <sub></sub><i><sub>3 xyz</sub></i>3


3 1 1


3 27


<i>xyz</i> <i>xyz</i>


   


áp dụng bất đẳng thức Côsi cho x+y ; y+z ; x+z ta có:


<i>x y</i><sub></sub>

 

. <i>y z</i><sub></sub>

 

. <i>z x</i><sub></sub>

<sub></sub>33

<i>x y</i><sub></sub>

 

. <i>y z</i><sub></sub>

 

. <i>x z</i><sub></sub>



 

 



3


2 3

<i>x y</i>

.

<i>y z</i>

.

<i>z x</i>





DÊu b»ng x¶y ra khi x=y=z=1


3


VËy S  8 1. 8



27 27 729


VËy S cã giá trị lớn nhất là 8


729 khi x=y=z=
1


3.


Bài 2:


<b>Giải</b>


áp dụng BĐT Bunhiacốpski cho 6 số (x,y,z) ;(x,y,z)


Ta cã

<sub></sub>

<i><sub>xy yz zx</sub></i><sub></sub> <sub></sub>

<sub></sub>

2 <sub></sub>

<i><sub>x</sub></i>2 <sub></sub> <i><sub>y</sub></i>2<sub></sub><i><sub>z</sub></i>2

2


<sub></sub>

<sub>1</sub>

<sub></sub>

<i><sub>x</sub></i>

2

<sub></sub>

<i><sub>y</sub></i>

2

<sub></sub>

<i><sub>z</sub></i>

2

2


áp dụng BĐT Bunhiacốpski cho (<i><sub>x y z</sub></i>2<sub>,</sub> 2<sub>,</sub> 2<sub>) vµ (1,1,1)</sub>
Ta cã:


2 2 2 2 2 2 2 4 4 4


2 2 2 2 4 4 4


( ) (1 1 1 )( )


( ) 3( )



<i>x</i> <i>y</i> <i>z</i> <i>x</i> <i>y</i> <i>z</i>


<i>x</i> <i>y</i> <i>z</i> <i>x</i> <i>y</i> <i>z</i>


      


     


Tõ (1) vµ (2) <sub></sub> 1 3(<sub></sub> <i>x</i>4 <sub></sub><i>y</i>4 <sub></sub><i>z</i>4)


4 4 4

1



3



<i>x</i>

<i>y</i>

<i>z</i>





</div>
<span class='text_page_counter'>(71)</span><div class='page_container' data-page=71>

VËy <i><sub>x</sub></i>4 <i><sub>y</sub></i>4 <i><sub>z</sub></i>4


  cã giá trị nhỏ nhất là 1


3 khi x=y=z=
3
3


Bài 3:


<b>Gi¶i</b>



A = x2<sub> – 2xy + 3y</sub>2<sub> – 2x – 10y + 20</sub>


= (x- y -1 )2<sub> + 2(y- 3)</sub>2<sub> + 1</sub>


mµ (x- y -1 )2<sub> ≥ 0 vµ (y- 3)</sub>2<sub> ≥ 0 </sub>


vËy x2<sub> – 2xy + 3y</sub>2<sub> – 2x – 10y + 20 ≥ 1 </sub>


dÊu ‘ =’ x¶y ra <=>













0
3


0
1


<i>y</i>
<i>y</i>


<i>x</i>


<=>








3
4


<i>y</i>
<i>x</i>


VËy Amin =1 khi x= 4 , y = 3


Bài 4:


<b>Giải</b>


Có D =


1
1


2
2







<i>x</i>
<i>x</i>


<i>x</i> <sub> xác định với mọi x và luôn dơng</sub>


Ta cã: A(x2<sub>+1) = x</sub>2<sub> +x +1</sub>


<=> (A-1)x2<sub> - x +A -1 =0</sub>


+ NÕu A=1 th× x =0 và ngợc lại


+ Nu A1 mun tn ti x thì điều kiện cần và đủ: 1- 4(A-1)2  0


4A2<sub> -8A + 3 </sub><sub></sub><sub>0 <=> </sub>


2
1


A 


2
3


VËy Amax=


2


3


Amin=


2
1


(Khi x=1) (khi x=-1)
Bài 5:


<b>Giải</b>


Gọi a là số tự nhiên cần tìm


Ta có a= 29 q +5 =31 q’<sub> =28 (q, q</sub>, <i><sub>N</sub></i>


 )


23
2


)
(


29 , ,







 <i>q</i> <i>q</i> <i>q</i>


,


<i>q</i>
<i>q </i>


 lµ sè lỴ ; q-q, <sub></sub><sub>1</sub>


A nhỏ nhất khi q,<sub> nhỏ nhất. Lúc đó</sub>


</div>
<span class='text_page_counter'>(72)</span><div class='page_container' data-page=72>

Hay q-q,<sub> nhá nhÊt</sub>


q-q, <sub>=1 vµ </sub><sub>2q</sub>, <sub>= 29-23 =6</sub>


VËy : q,<sub> =3 vµ a =121 là số cần tìm</sub>


<b>II - Giải phơng trình và hệ phơng trình:</b>


Bài 1:


<b>Giải</b>


áp dụng : Nếu a, b > 0 th× :  2


<i>a</i>
<i>b</i>
<i>b</i>
<i>a</i>



(2)  (3 2 1)(3<i>x</i>2<i>y</i><i>z</i>)36
<i>z</i>


<i>y</i>
<i>x</i>


 6(  )3(  )2( )22
<i>y</i>
<i>z</i>
<i>z</i>
<i>y</i>
<i>x</i>


<i>z</i>
<i>z</i>
<i>x</i>
<i>x</i>
<i>y</i>
<i>y</i>
<i>x</i>


Mặt khác : vì x, y, z > nªn 6(  )12
<i>x</i>
<i>y</i>
<i>y</i>
<i>x</i>


Vµ 3(  )6


<i>x</i>


<i>z</i>
<i>z</i>
<i>x</i>


; 2(  )4
<i>z</i>
<i>y</i>
<i>y</i>
<i>z</i>


Nªn (  )3(  )2(  )22
<i>y</i>
<i>z</i>
<i>z</i>
<i>y</i>
<i>x</i>


<i>z</i>
<i>z</i>
<i>x</i>
<i>x</i>
<i>y</i>
<i>y</i>
<i>x</i>


Dấu '' = '' xảy ra khi x = y = z , thay vào (1) ta đợc:
x + x2<sub> + x</sub>3<sub> = 14 <=> (x - 2)(x</sub>2<sub> + 3x + 7) = 0</sub>


<=> x - 2 = 0 <=> x = 2



Vậy hệ phơng trình có nghiệm duy nhÊt : x = y = z = 2
Bµi 2:


<b>Gi¶i</b>


Ta cã <sub>3</sub><i><sub>x</sub></i>2 <sub>6</sub><i><sub>x</sub></i> <sub>19</sub>


  3.(<i>x</i>2 2<i>x</i>1) 16


<sub>3.(</sub><i><sub>x</sub></i><sub>1)</sub>2<sub>16 16</sub>


<sub>5</sub><i><sub>x</sub></i>2 <sub>10</sub><i><sub>x</sub></i> <sub>14 5.</sub>

<sub></sub>

<i><sub>x</sub></i> <sub>1</sub>

<sub></sub>

2 <sub>9 9</sub>


     


VËy <sub>4. 3</sub><i><sub>x</sub></i>2 <sub>6</sub><i><sub>x</sub></i> <sub>19</sub> <sub>5</sub><i><sub>x</sub></i>2 <sub>10</sub><i><sub>x</sub></i> <sub>14 2 3 5</sub>


       


DÊu ( = ) x¶y ra khi x+1 = 0  x = -1


VËy <sub>4 3</sub><i><sub>x</sub></i>2 <sub>6</sub><i><sub>x</sub></i> <sub>19</sub> <sub>5</sub><i><sub>x</sub></i>2 <sub>10</sub><i><sub>x</sub></i> <sub>14 4 2</sub><i><sub>x x</sub></i>2


        khi x = -1


Vậy phơng trình có nghiệm duy nhất x = -1
Bài 3:


</div>
<span class='text_page_counter'>(73)</span><div class='page_container' data-page=73>

<b>Giải</b>



áp dụng BĐT BunhiaCốpski ta có:




2 2 2 2 2


2 1 1 . 2 2. 2 2


<i>x</i>  <i>x</i>   <i>x</i>   <i>x</i> 


Dấu (=) xảy ra khi x = 1


Mặt khác 4<i>y</i>2 4<i>y</i> 3

<sub></sub>

2<i>y</i>1

<sub></sub>

2  2 2


DÊu (=) x¶y ra khi y = -1


2


VËy <i><sub>x</sub></i> <sub>2</sub> <i><sub>x</sub></i>2 <sub>4</sub><i><sub>y</sub></i>2 <sub>4</sub><i><sub>y</sub></i> <sub>3 2</sub>


      khi x =1 vµ y =-1<sub>2</sub>


Vậy nghiệm của phơng trình là


1
1
2


<i>x</i>
<i>y</i>












.


Bài 4:


<b>Giải</b>


áp dụng BĐT Cosi ta cã:


4 4 4 4 4 4


4 4 4


2 2 2 2 2 2


2 2 2 2 2 2 2 2 2 2 2 2


x


2 2 2



2 2 2


<i>x</i> <i>y</i> <i>y</i> <i>z</i> <i>z</i> <i>x</i>


<i>y</i> <i>z</i>


<i>x y</i> <i>y z</i> <i>z x</i>


<i>x y</i> <i>y z</i> <i>z y</i> <i>z z</i> <i>x z</i> <i>y x</i>


  


    


  


  


  


2 2 2


.( )


<i>y xz z xy x yz</i>
<i>xyz x y z</i>


  


  



V× x+y+z = 1 Nªn <i><sub>x</sub></i>4 <i><sub>y</sub></i>4 <i><sub>z</sub></i>4 <i><sub>xyz</sub></i>


  


DÊu (=) x¶y ra khi x = y = z =1


3


VËy <sub>4</sub> <i>x y z</i><sub>4</sub> <sub>4</sub> 1


<i>x</i> <i>y</i> <i>z</i> <i>xyz</i>


  




  




cã nghiÖm x = y = z =1


3


Bµi 5:


</div>
<span class='text_page_counter'>(74)</span><div class='page_container' data-page=74>

Tõ phơng trình (1) <sub>8</sub> <i><sub>y</sub></i>2 <sub>0</sub>



   hay <i>y </i> 8


Từ phơng trình (2) <i>x</i>2 2 <i>x y</i>. 2 2 <i>x</i>




2 2


2


2 2 2 0


( 2) 0


2


2


<i>x</i> <i>x</i>


<i>x</i>
<i>x</i>
<i>x</i>


   


  


 



 


NÕu x = <sub>2</sub> th× y = 2 <sub>2</sub>, nÕu x = - <sub>2</sub> thì y = -2 <sub>2</sub>


Vậy hệ phơng trình có nghiệm 2
2


<i>x</i>
<i>y</i>


 <sub></sub>









vµ 2 2


2 2
<i>x</i>


<i>y</i>


 <sub></sub>










.


<b>III </b>–<b> T×m nghiƯm nguyên của phơng trình:</b>
<b>Giải</b>


Không mất tính tổng quát ta giả sö <i>x</i> <i>y z</i>


Ta cã 2 1 1 1 3 2<i>z</i> 3


<i>x</i> <i>y</i> <i>z</i> <i>z</i>




Mà z nguyên dơng vậy z = 1


Thay z = 1 vào phơng trình ta đợc 1 1 1


<i>x</i> <i>y</i> 


Theo gi¶ sư xy nªn 1 = 1 1


<i>x</i> <i>y</i>


1



<i>y</i>


  <i>y</i>2 mà y nguyên dơng
Nên y = 1 hoặc y = 2


Với y = 1 không thích hợp
Với y = 2 ta cã x = 2


VËy (2 ,2,1) lµ một nghiệm của phơng trình


Hoỏn v cỏc s trờn ta đợc các nghiệm của phơng trình là (2,2,1) ; (2,1,2) ; (1,2,2).


</div>
<span class='text_page_counter'>(75)</span><div class='page_container' data-page=75>

p


<b>Phần V - Bài tập tự gi¶i</b>


1. Chứng minh các bất đẳng thức sau:


a)

x

2

<sub></sub>

y

2

<sub></sub>

z

2

<sub></sub>

xy yz zx

<sub></sub>

<sub></sub>



b) <sub>x</sub>2<sub></sub><sub>y</sub>2<sub></sub><sub>z</sub>2 <sub></sub><sub>2xy 2yz 2zx</sub><sub></sub> <sub></sub>
c) x2y2z23 2(x y z)  


d) x2y2z2t2k2 x(y z t k)  


e)


2 2 2


2



x y z x y z


( )


3 3


   




2. Cho a<sub>1</sub> a<sub>2</sub> a<sub>3</sub> vµ b<sub>1</sub> b<sub>2</sub> b<sub>3</sub>


a) a1 a b2. 1 b2 a b1 1 a b2 2


2 2 2


  




b) a1 a2 a b3. 1 b2 b3 a b1 1 a b2 2 a b3 3


3 3 3


     




¸p dông: <sub>(x y)(x</sub> 3 <sub>y )(x</sub>3 7 <sub>y</sub>7) <sub>4(x</sub>11<sub>y )</sub>11 Víi x,y >0


3Cho a,b,c,d 0 Tho· a c d, b c+d  


Chøng minh: <sub>ab ad bc</sub><sub></sub> <sub></sub>


4. Cho a,b,c >0 tho· a2 b2 c2 5
3


</div>
<span class='text_page_counter'>(76)</span><div class='page_container' data-page=76>

1 1 1 1
a b c  abc


5.Cho 0 a,b,c 1  . Chøng minh r»ng:


3 3 3 2 2 2


2a 2b 2c  3 a b b c c a  .


6. Chøng minh r»ng:


a) <sub>(a c)</sub><sub></sub> 2<sub></sub><sub>(b d)</sub><sub></sub> 2 <sub></sub> <sub>a</sub>2<sub></sub><sub>b</sub>2 <sub></sub> <sub>c</sub>2<sub></sub><sub>d</sub>2


b) 1 <sub>2</sub> 1 <sub>2</sub> 2


1 x 1 y 1 xy Víi (xy 1)
7. Cho 0 < a < b < c. Chøng minh r»ng:


a) a (b3 2 c ) b (c2  3 2 a ) c (a2  3 2 b ) 02 


b) b(1 1) 1(a c) (a c)(1 1)


a b  b    a c



8. Cho a,b,c là độ dài 3 cạnh của tam giác.Chứng minh rằng:
a) a2b2c2 2(ab bc ca) 


b) abc (a b c)(b c a)(c a b)      


c) a3b3 c32abc a (b c) b (c a) c (a b) 2   2   2 


d) a(b c) 2b(c a) 2c(a b) 2 a3b3c3


e) <sub>2a b</sub>2 2<sub></sub><sub>2b c</sub>2 2<sub></sub><sub>2c a</sub>2 2<sub></sub> <sub>a</sub>4 <sub></sub> <sub>b</sub>4 <sub></sub> <sub>c</sub>4 <sub></sub> <sub>0</sub>
9. Cho 0 a,b,c,d 1  Chøng minh r»ng:


2 2 2 2 2


(a b c d)   4(a b c d )


10. Cho a1, a2 >0 vµ a c<sub>1 1</sub> b , a b<sub>1</sub>2 <sub>2 2</sub> b<sub>2</sub>2 .Chøng minh r»ng:


((a<sub>1</sub>a )(c<sub>2</sub> <sub>1</sub>c ) (b<sub>2</sub>  <sub>1</sub>b )<sub>2</sub> 2


11. Cho a, b, c >0. Chøng minh:


a b c 3


b c c a a b     2


12. a,b,c lµ 3 cạnh của một tam giác. Chứng minh:


a b c



3
b c a a c b a b c        


</div>
<span class='text_page_counter'>(77)</span><div class='page_container' data-page=77>

13. Cho a, b, c, d >0 vµ abcd=1. Chøng minh r»ng:


2 2 2 2


a b c d a(b c) b(c d) d(c a) 10     


14. Cho tam giác ABC. Lấy điểm M ở trong tam giác, các đờng thẳng AM, BM,
CM cắt các cạnh của tam giác lần lợt A1, B1, C1. Chứng minh rằng:




1 1 1


AM BM CM


6
A M  B M  C M 
15. Cho a,b>0 vµ x+y=1. Chøng minh r»ng:


2 2


2 3


14
ab a b 



16. Chứng minh điều kiện cần và đủ để các số a, b, c cùng dấu là


ab+bc+ca>0 vµ 1 1 1 0
ab bc ca  


17. Chứng minh rằng khơng tồn tại 3 số x,y,z đồng thời thỗ mãn ba bất đẳng thức


sau: x  y z , y  z x , z  x y


18. Cho 0<a,b,c<1. Chứng minh rằng có ít nhất một trong các bất đẳng thức sau là


sai: a(1 b) 1, b(1 c) 1, c(1 a) 1


4 4 4


     


19. Cho 4 sè a,b,c,d tho· m·n ®iỊu kiƯn ac 2(b d) 


Chứng minh rằng có ít nhất một trong các bất đẳng thức sau là sai:


2 2


a 4b, c 4d


20. Chứng minh rằng một trong các bất đẳng thức sau là đúng:


2 2 1 2 2 2 1 2 2 2 1 2


a b (b c) , b c (c a) , c a (a b)



2 2 2


        


21. Cho a, b >0 . Chøng minh


1 a b a b a b


( )


2 1 a 1 b 1 a b 1 a 1 b




   


     


22. Cho a, b, c >0 . Chøng minh


a b c d


A


a b c b c d c d a d a b


   


       



</div>
<span class='text_page_counter'>(78)</span><div class='page_container' data-page=78>

23. Cho a ,b ,c là độ dài của 3 cạnh của một tam giác . Chứng minh rằng:


a b c


1 2


b c c a a b


   


  


24.Cho n lµ số nguyên dơng, chứng minh:


2


1 1 1


... 1


n n 1    n 
25. Chøng minh r»ng:


a) 5 1 ... <sub>2</sub> 1 <sub>2</sub> 1


11 13   n (n 1)  2 Víi n 1


b) 1 1 ... 1 <sub>2</sub> 1



9 16  (2n 1)  4 Víi n 1


c) 2( n 1 1) 1 1 ... 1 2 n 1(n 2)


2 n


        


26. Cho an <sub>(2n 1)( n</sub>2 <sub>n 1)</sub>(n 1,2,...)


  


Chøng minh r»ng: a<sub>1</sub> a<sub>2</sub> ... a<sub>n</sub> n
n 2


   




27. Cho a>b>0. Chøng minh r»ng: a 1 3
b(a b)


 



28. Cho a,b,c >0, a+b+c=1. Chøng minh r»ng:


1 1 1


(1 )(1 )(1 ) 64



a b c


   


29. Cho a , a ,a ,a , a >0 <sub>1</sub> <sub>2</sub> <sub>3</sub> <sub>4</sub> <sub>5</sub>
Cã tæng b»ng 1. Chøng minh r»ng:


1 2 3 4 5


1 1 1 1 1


( 1)( 1)( 1)( 1)( 1) 1024


a  a  a  a  a  


30. Cho h×nh thang ABCD cã AD//BC cã diện tích S. Gọi E là giao điểm của hai


đ-ờng chéo. Chứng minh rằng S<sub>ABE</sub> 1S
4


31. Cho a,b,c là 3 cạnh của tam giác có chu vi 2p. Chứng minh r»ng:


</div>
<span class='text_page_counter'>(79)</span><div class='page_container' data-page=79>

p  p a  p b  p c  3p


32. Cho 3x-4y =7 .Chøng minh r»ng: 3x -4y 2 2 7


Hớng dẫn – Gợi ý BT tự giải
Sử dụng phơng pháp định nghĩa và phép biến đổi tơng đơng



1. a) (x y) 2(y z) 2(z x) 2 0


b) (x y z)  2 0


c) (x 1) 2(y 1) 2(z 1) 2 0


d) (x y)2 (x z)2 (x t)2 (x k)2 0


2   2  2  2 


e) (x y) 2(y z) 2(z x) 2 0


2. Sử dụng phơng pháp định nghĩa và phép biến đổi tơng đơng


a) 1(a b<sub>1 1</sub> a b )<sub>2 2</sub> 1(a<sub>1</sub> a ) (b<sub>2</sub> 1 <sub>1</sub> b )<sub>2</sub> 1(a<sub>1</sub> a )(b<sub>2</sub> <sub>1</sub> b ) 0<sub>2</sub>


2   2  2  4   


b)(a<sub>1</sub> a )(b<sub>2</sub> <sub>1</sub> b ) (a<sub>2</sub>  <sub>2</sub> a )(b<sub>3</sub> <sub>2</sub> b ) (a<sub>3</sub>  <sub>1</sub> a )(b<sub>3</sub> <sub>1</sub> b ) 0<sub>3</sub> 


3. Sử dụng phơng pháp định nghĩa và phép biến đổi tơng đơng


a c d,b d c     (a c)(b d) cd    điều phải chứng minh.
4. Sử dụng phơng pháp định nghĩa và phép biến đổi tơng đơng


2 1 2 2 2 5


(a b c) 0 ac bc ab (a b c ) 1



2 6


          


5. Sử dụng phơng pháp định nghĩa và phép biến đổi tơng đơng


2 2 2 3 3


(1 a )(1 b) 1 a b a     b a b


Tơng tự:

1 b c b

2

3

c ,1 c a c

3

2

2

a

3.cộng lại đợc điều phải chứng


minh


6. Sử dụng phơng pháp định nghĩa và phép biến đổi tơng đơng


a)ac bd  (a2b )(c2 2d )2 đúng


</div>
<span class='text_page_counter'>(80)</span><div class='page_container' data-page=80>

7 Sử dụng phơng pháp định nghĩa và phép biến đổi tơng đơng và các
a)(a b)(c a)(c b)(ab bc ca) 0      vì (<sub>0 a b c</sub><sub></sub> <sub></sub> <sub></sub> );


b)(a c)(b c)(b a) 0   


8. Sử dụng phơng pháp định nghĩa và phép biến đổi tơng đơng và các
Bất đẳng thức trong tam giác.


a)a2  a(b c) , b2  b(b c), c 2  c(c a) cộng lại đợc điều phải chứng minh;


b)



2
2
2


(a b c)(b c a) b


(b c a)(c a b) c


(c a b)(a b c) a


    


    


    


Nhân các vế bất đẳng thức trên ta đợc điều phải chứng minh
c) (<i>a b c a b c a c b</i>  )(   )(   ) 0


d)(<i>a b c b c a a c b</i>  )(   )(   ) 0


e) (<i>a b c a b c a c b b c a</i>  )(   )(   )(   ) 0


9.áp dụng Bất đẳng thức phụ


¸p dơng (<i>x y</i> )2 4<i>xy</i> và<i>x x khi</i> 2 0 <i>x</i> 1


10. áp dụng (<i>x y</i> )2 4<i>xy</i>


11. .áp dụng Bất đẳng thức phụ


Chia ra 3 bất đẳng thức nhỏ sau đó:


Céng vÕ theo vế và áp dụng (<i>x y z</i>)(1 1 1) 9 ( , ,<i>x y z</i> 0)


<i>x</i> <i>y</i> <i>z</i>


     


12. áp dụng phơng pháp đổi biến số


§Ỉt <i>x b c a y a c b z a b c</i>   ,    , và áp dụng <i>x</i> <i>y</i> 2


<i>y</i>  <i>x</i>  víi <i>x y </i>, 0


13. Ta cã <i>cd</i> 1


<i>ab</i>


 nªn <i>a</i>2 <i>b</i>2 <i>c</i>2 <i>d</i>2 2(<i>ab cd</i>) 2(<i>ab</i> 1 ) 4


<i>ab</i>


       


Vµ <i>a b c</i>( ) <i>b c d</i>( ) <i>d c a</i>( ) (<i>ab</i> 1 ) (<i>ac</i> 1 ) (<i>bc</i> 1 ) 6


<i>ab</i> <i>ac</i> <i>bc</i>


           



</div>
<span class='text_page_counter'>(81)</span><div class='page_container' data-page=81>

14. Gäi <i>S S S S</i>, ,<sub>1</sub> <sub>2</sub>, <sub>3</sub> lÇn lợt là diện tích các tâm giác ABC, MBC, MAC, MAB. Ta
cã:


1 1 1 2 3


1 1 1 1 1 1


<i>AA</i> <i>S</i> <i>AM</i> <i>AA</i> <i>MA</i> <i>S S</i> <i>S</i> <i>S</i>


<i>A M</i> <i>S</i> <i>A M</i> <i>A M</i> <i>S</i> <i>S</i>


  




Tơng tự cho
1


<i>BM</i>


<i>B M</i> và <sub>1</sub>
<i>CM</i>


<i>C M</i> Sau đó áp dụng


1
2


<i>x</i>
<i>x</i>



  (x > 0).


15. 2 <sub>2</sub> 3 <sub>2</sub> 4 <sub>2</sub> 3 <sub>2</sub> 1 2( 1 <sub>2</sub> 1 <sub>2</sub>)


2 2 2


<i>ab a</i> <i>b</i>  <i>ab a</i> <i>b</i>  <i>ab</i> <i>ab a</i> <i>b</i>


¸p dơng (<i>x y</i> )2 4<i>xy</i> vµ 1 1 4 ( ,<i>x y</i> 0)


<i>x</i>  <i>y</i> <i>x y</i> 
16. Sư dơng phơng pháp chứng minh phản chứng
Xét hai trờng hợp


TH1: <i>abc </i>0 th× a > 0, b > 0, c > 0
TH2: abc < 0 th× a < 0, b < 0, c < 0.


17. Bình phơng hai vế của bất đăng thức đã cho, chuyển về vế trái rồi nhân lại


18. 0 (1 ) 1 ( 1) 1


4 2 4


<i>a</i> <i>a</i> <i>a</i>


     


19. <i>a</i>2 <i>c</i>2 4(<i>b d</i> ) 2 <i>ac</i> Vô lý



20. Giả sử 2 2 1<sub>(</sub> <sub>) ,</sub>2 2 2 1<sub>(</sub> <sub>) ,</sub>2 2 2 1<sub>(</sub> <sub>)</sub>2


2 2 2


<i>a</i> <i>b</i>  <i>b c</i> <i>b</i> <i>c</i>  <i>c a</i> <i>c</i> <i>a</i>  <i>c a</i>


Cộng lại ta đợc: (<i>a b</i> )2 (<i>b c</i> )2 (<i>c a</i> )2 0, vơ lý !
21. Sử dụng các tính chất đặc biệt của tỉ lệ thức.


,


1 1 1 1


<i>a</i> <i>a b</i> <i>b</i> <i>a b</i>


<i>a</i> <i>a b</i> <i>b</i> <i>a b</i>


 


 


     


22. Sử dụng các tính chất đặc biệt của tỉ lệ thức


.


<i>a</i> <i>a</i> <i>a d</i>


<i>a b c d</i> <i>a b c</i> <i>a b c d</i>







</div>
<span class='text_page_counter'>(82)</span><div class='page_container' data-page=82>

23. Tơng tự bài 22: <i>a</i> <i>a</i> <i>a a</i>


<i>a b c b c</i> <i>a b c</i>








24. Sử dụng phơng pháp làm tréi


2


2 2 2 2


1 1 1 1 1 1 1


... ... 1


1


<i>n</i> <i>n</i>


<i>n n</i> <i>n</i> <i>n n</i> <i>n</i> <i>n</i> <i>n</i>





         




25. Sö dụng phơng pháp làm trội


a) <sub>2</sub> 1 <sub>2</sub> <sub>2</sub> 1 1 1( 1 );


( 1) 2 2 1 2 1


<i>k k</i>   <i>k</i>  <i>k</i>   <i>k</i>  <i>k</i> 


b) 1 <sub>2</sub> <sub>2</sub> 1 1 1 1( 1 );


(2<i>k</i> 1) 4<i>k</i> 4<i>k</i> 1 4 ( <i>k k</i>1) 4 <i>k</i>  <i>k</i> 1


c) 1 1 1 1 2( 1 )


2 <i>k</i>1 <i>k</i>   <i>k</i>  2 <i>k</i>  <i>k</i>  <i>k</i>   <i>k</i> vµ


1 2( 1 )


1 <i>k</i> <i>k</i>


<i>k</i>    


26. Sö dụng phơng pháp làm trội



2


2( 1 ) 1 1


1


4 4 1


<i>k</i>


<i>k</i> <i>k</i>


<i>a</i>


<i>k</i> <i>k</i>


<i>k</i> <i>k</i>


 


  




 


27. Sö dơng B§T Cosi


1 1



( ) 3


( ) ( )


<i>a</i> <i>a b</i> <i>b</i>


<i>b a b</i> <i>b a b</i>


     


 


28. Sư dơng B§T Cosi


4
2
1


1 <i>a b c</i> 1 1 <i>b</i> <i>c</i> 4 <i>bc</i>


<i>a</i>


<i>a</i> <i>a</i> <i>a a</i> <i>a</i>


 


       


29. Sö dơng B§T Cosi



4


2 3 4 5


1 2 3 4 5 2 3 4 5


1 1 1 1 1 1 1


4
1


1 <i>a</i> <i>a</i> <i>a</i> <i>a</i> <i>a</i> 1 <i>a</i> <i>a</i> <i>a</i> <i>a</i> <i>a a a a</i>


<i>a</i> <i>a</i> <i>a</i> <i>a</i> <i>a</i> <i>a</i> <i>a</i>


   


       


30. Sử dụng BĐT Cosi


Chỉ cần chứng minh <i>S</i>2<i><sub>ABE</sub></i> <i>S<sub>CEB</sub></i>.<i>S<sub>AED</sub></i> và


4 <sub>2</sub> <sub>2</sub>


4,


( )



<i>ABE</i>


<i>S</i> <i>x y</i>


<i>S</i> <i>x y</i>


 




 




 


víi x = BC, y = AD


</div>
<span class='text_page_counter'>(83)</span><div class='page_container' data-page=83>

31. Sư dơng B§T Bunhiacopsky
2


(1. <i>p a</i> 1. <i>p b</i> 1. <i>p c</i> ) 3<i>p</i>


32. Sư dơng B§T Bunhiacopsky


2 2 2 2


</div>

<!--links-->
chuyên đề bồi dưỡng HSG THCS ( Các dạng toán dung dịch )
  • 10
  • 3
  • 43
  • ×